You are on page 1of 32

NCLEX Exam: Obstetrical Nursing – Antepartum (50 3. “It is the fetal movement that is felt by the mother.


Items) 4. “It is the thinning of the lower uterine segment.”
9. A nurse midwife is performing an assessment of a
1. A nursing instructor is conducting lecture and is pregnant client and is assessing the client for the
reviewing the functions of the female reproductive presence of ballottement. Which of the following would
system. She asks Mark to describe the follicle- the nurse implement to test for the presence of
stimulating hormone (FSH) and the luteinizing ballottement?
hormone (LH). Mark accurately responds by stating 1. Auscultating for fetal heart sounds
that: 2. Palpating the abdomen for fetal movement
1. FSH and LH are released from the anterior pituitary gland. 3. Assessing the cervix for thinning
2. FSH and LH are secreted by the corpus luteum of the ovary 4. Initiating a gentle upward tap on the cervix
3. FSH and LH are secreted by the adrenal glands 10. A nurse is assisting in performing an assessment on
4. FSH and LH stimulate the formation of milk during a client who suspects that she is pregnant and is
pregnancy. checking the client for probable signs of
2. A nurse is describing the process of fetal circulation pregnancy. Select all probable signs of pregnancy.
to a client during a prenatal visit. The nurse accurately 1. Uterine enlargement
tells the client that fetal circulation consists of: 2. Fetal heart rate detected by nonelectric device
1. Two umbilical veins and one umbilical artery 3. Outline of the fetus via radiography or ultrasound
2. Two umbilical arteries and one umbilical vein 4. Chadwick’s sign
3. Arteries carrying oxygenated blood to the fetus 5. Braxton Hicks contractions
4. Veins carrying deoxygenated blood to the fetus 6. Ballottement
3. During a prenatal visit at 38 weeks, a nurse assesses 11. A pregnant client calls the clinic and tells a nurse
the fetal heart rate. The nurse determines that the fetal that she is experiencing leg cramps and is awakened by
heart rate is normal if which of the following is noted? the cramps at night. To provide relief from the leg
1. 80 BPM cramps, the nurse tells the client to:
2. 100 BPM 1. Dorsiflex the foot while extending the knee when the
3. 150 BPM cramps occur
4. 180 BPM 2. Dorsiflex the foot while flexing the knee when the cramps
4. A client arrives at a prenatal clinic for the first occur
prenatal assessment. The client tells a nurse that the 3. Plantar flex the foot while flexing the knee when the
first day of her last menstrual period was September cramps occur
19th, 2013. Using Naegele’s rule, the nurse determines 4. Plantar flex the foot while extending the knee when the
the estimated date of confinement as: cramps occur.
1. July 26, 2013 12. A nurse is providing instructions to a client in the
2. June 12, 2014 first trimester of pregnancy regarding measures to
3. June 26, 2014 assist in reducing breast tenderness. The nurse tells
4. July 12, 2014 the client to:
5. A nurse is collecting data during an admission 1. Avoid wearing a bra
assessment of a client who is pregnant with twins. The 2. Wash the nipples and areola area daily with soap, and
client has a healthy 5-year old child that was delivered massage the breasts with lotion.
at 37 weeks and tells the nurse that she doesn’t have 3. Wear tight-fitting blouses or dresses to provide support
any history of abortion or fetal demise. The nurse 4. Wash the breasts with warm water and keep them dry
would document the GTPAL for this client as: 13. A pregnant client in the last trimester has been
1. G = 3, T = 2, P = 0, A = 0, L =1 admitted to the hospital with a diagnosis of severe
2. G = 2, T = 0, P = 1, A = 0, L =1 preeclampsia. A nurse monitors for complications
3. G = 1, T = 1. P = 1, A = 0, L = 1 associated with the diagnosis and assesses the client
4. G = 2, T = 0, P = 0, A = 0, L = 1 for:
6. A nurse is performing an assessment of a primipara 1. Any bleeding, such as in the gums, petechiae, and purpura.
who is being evaluated in a clinic during her second 2. Enlargement of the breasts
trimester of pregnancy. Which of the following 3. Periods of fetal movement followed by quiet periods
indicates an abnormal physical finding necessitating 4. Complaints of feeling hot when the room is cool
further testing? 14. A client in the first trimester of pregnancy arrives
1. Consistent increase in fundal height at a health care clinic and reports that she has been
2. Fetal heart rate of 180 BPM experiencing vaginal bleeding. A threatened abortion is
3. Braxton hicks contractions suspected, and the nurse instructs the client regarding
4. Quickening management of care. Which statement, if made by the
7. A nurse is reviewing the record of a client who has client, indicates a need for further education?
just been told that a pregnancy test is positive. The 1. “I will maintain strict bedrest throughout the remainder of
physician has documented the presence of a Goodell’s pregnancy.”
sign. The nurse determines this sign indicates: 2. “I will avoid sexual intercourse until the bleeding has
1. A softening of the cervix stopped, and for 2 weeks following the last evidence of
2. A soft blowing sound that corresponds to the maternal bleeding.”
pulse during auscultation of the uterus. 3. “I will count the number of perineal pads used on a daily
3. The presence of hCG in the urine basis and note the amount and color of blood on the pad.”
4. The presence of fetal movement 4. “I will watch for the evidence of the passage of tissue.”
8. A nursing instructor asks a nursing student who is 15. A prenatal nurse is providing instructions to a
preparing to assist with the assessment of a pregnant group of pregnant client regarding measures to prevent
client to describe the process of quickening. Which of toxoplasmosis. Which statement if made by one of the
the following statements if made by the student clients indicates a need for further instructions?
indicates an understanding of this term? 1. “I need to cook meat thoroughly.”
1. “It is the irregular, painless contractions that occur 2. “I need to avoid touching mucous membranes of the
throughout pregnancy.” mouth or eyes while handling raw meat.”
2. “It is the soft blowing sound that can be heard when the 3. “I need to drink unpasteurized milk only.”
uterus is auscultated.”
4. “I need to avoid contact with materials that are possibly 3. Seizures do not occur
contaminated with cat feces.” 4. Scotomas are present
16. A homecare nurse visits a pregnant client who has 24. A nurse is caring for a pregnant client with severe
a diagnosis of mild Preeclampsia and who is being preeclampsia who is receiving IV magnesium sulfate.
monitored for pregnancy induced hypertension (PIH). Select all nursing interventions that apply in the care
Which assessment finding indicates a worsening of the for the client.
Preeclampsia and the need to notify the physician? 1. Monitor maternal vital signs every 2 hours
1. Blood pressure reading is at the prenatal baseline 2. Notify the physician if respirations are less than 18 per
2. Urinary output has increased minute.
3. The client complains of a headache and blurred vision 3. Monitor renal function and cardiac function closely
4. Dependent edema has resolved 4. Keep calcium gluconate on hand in case of a magnesium
17. A nurse implements a teaching plan for a pregnant sulfate overdose
client who is newly diagnosed with gestational 5. Monitor deep tendon reflexes hourly
diabetes. Which statement if made by the client 6. Monitor I and O’s hourly
indicates a need for further education? 7. Notify the physician if urinary output is less than 30 ml per
1. “I need to stay on the diabetic diet.” hour.
2. “I will perform glucose monitoring at home.” 25. In the 12th week of gestation, a client completely
3. “I need to avoid exercise because of the negative effects of expels the products of conception. Because the client is
insulin production.” Rh negative, the nurse must:
4. “I need to be aware of any infections and report signs of 1. Administer RhoGAM within 72 hours
infection immediately to my health care provider.” 2. Make certain she receives RhoGAM on her first clinic visit
18. A primigravida is receiving magnesium sulfate for 3. Not give RhoGAM, since it is not used with the birth of a
the treatment of pregnancy induced hypertension stillborn
(PIH). The nurse who is caring for the client is 4. Make certain the client does not receive RhoGAM, since the
performing assessments every 30 minutes. Which gestation only lasted 12 weeks.
assessment finding would be of most concern to the 26. In a lecture on sexual functioning, the nurse plans
nurse? to include the fact that ovulation occurs when the:
1. Urinary output of 20 ml since the previous assessment 1. Oxytocin is too high
2. Deep tendon reflexes of 2+ 2. Blood level of LH is too high
3. Respiratory rate of 10 BPM 3. Progesterone level is high
4. Fetal heart rate of 120 BPM 4. Endometrial wall is sloughed off.
19. A nurse is caring for a pregnant client with 27. The chief function of progesterone is the:
Preeclampsia. The nurse prepares a plan of care for the 1. Development of the female reproductive system
client and documents in the plan that if the client 2. Stimulation of the follicles for ovulation to occur
progresses from Preeclampsia to eclampsia, the nurse’s 3. Preparation of the uterus to receive a fertilized egg
first action is to: 4. Establishment of secondary male sex characteristics
1. Administer magnesium sulfate intravenously 28. The developing cells are called a fetus from the:
2. Assess the blood pressure and fetal heart rate 1. Time the fetal heart is heard
3. Clean and maintain an open airway 2. Eighth week to the time of birth
4. Administer oxygen by face mask 3. Implantation of the fertilized ovum
20. A nurse is monitoring a pregnant client with 4. End of the send week to the onset of labor
pregnancy induced hypertension who is at risk for 29. After the first four months of pregnancy, the chief
Preeclampsia. The nurse checks the client for which source of estrogen and progesterone is the:
specific signs of Preeclampsia (select all that apply)? 1. Placenta
1. Elevated blood pressure 2. Adrenal cortex
2. Negative urinary protein 3. Corpus luteum
3. Facial edema 4. Anterior hypophysis
4. Increased respirations 30. The nurse recognizes that an expected change in
21. Rho (D) immune globulin (RhoGAM) is prescribed the hematologic system that occurs during the
for a woman following delivery of a newborn infant and 2nd trimester of pregnancy is:
the nurse provides information to the woman about the 1. A decrease in WBC’s
purpose of the medication. The nurse determines that 2. In increase in hematocrit
the woman understands the purpose of the medication 3. An increase in blood volume
if the woman states that it will protect her next baby 4. A decrease in sedimentation rate
from which of the following? 31. The nurse is aware than an adaptation of pregnancy
1. Being affected by Rh incompatibility is an increased blood supply to the pelvic region that
2. Having Rh positive blood results in a purplish discoloration of the vaginal
3. Developing a rubella infection mucosa, which is known as:
4. Developing physiological jaundice 1. Ladin’s sign
22. A pregnant client is receiving magnesium sulfate 2. Hegar’s sign
for the management of preeclampsia. A nurse 3. Goodell’s sign
determines the client is experiencing toxicity from the 4. Chadwick’s sign
medication if which of the following is noted on 32. A pregnant client is making her
assessment? first Antepartum visit. She has a two year old son born
1. Presence of deep tendon reflexes at 40 weeks, a 5 year old daughter born at 38 weeks,
2. Serum magnesium level of 6 mEq/L and 7 year old twin daughters born at 35 weeks. She
3. Proteinuria of +3 had a spontaneous abortion 3 years ago at 10 weeks.
4. Respirations of 10 per minute Using the GTPAL format, the nurse should identify that
23. A woman with preeclampsia is receiving the client is:
magnesium sulfate. The nurse assigned to care for the 1. G4 T3 P2 A1 L4
client determines that the magnesium therapy is 2. G5 T2 P2 A1 L4
effective if: 3. G5 T2 P1 A1 L4
1. Ankle clonus in noted 4. G4 T3 P1 A1 L4
2. The blood pressure decreases
33. An expected cardiopulmonary adaptation 1. Mastitis
experienced by most pregnant women is: 2. Metabolic alkalosis
1. Tachycardia 3. Physiologic anemia
2. Dyspnea at rest 4. Respiratory acidosis
3. Progression of dependent edema 44. A 21-year old client, 6 weeks’ pregnant is
4. Shortness of breath on exertion diagnosed with hyperemesis gravidarum. This
34. Nutritional planning for a newly pregnant woman of excessive vomiting during pregnancy will often result
average height and weighing 145 pounds should in which of the following conditions?
include: 1. Bowel perforation
1. A decrease of 200 calories a day 2. Electrolyte imbalance
2. An increase of 300 calories a day 3. Miscarriage
3. An increase of 500 calories a day 4. Pregnancy induced hypertension (PIH)
4. A maintenance of her present caloric intake per day 45. Clients with gestational diabetes are usually
35. During a prenatal examination, the nurse draws managed by which of the following therapies?
blood from a young Rh negative client and explain that 1. Diet
an indirect Coombs test will be performed to predict 2. NPH insulin (long-acting)
whether the fetus is at risk for: 3. Oral hypoglycemic drugs
1. Acute hemolytic disease 4. Oral hypoglycemic drugs and insulin
2. Respiratory distress syndrome 46. The antagonist for magnesium sulfate should be
3. Protein metabolic deficiency readily available to any client receiving IV magnesium.
4. Physiologic hyperbilirubinemia Which of the following drugs is the antidote for
36. When involved in prenatal teaching, the nurse magnesium toxicity?
should advise the clients that an increase in vaginal 1. Calcium gluconate
secretions during pregnancy is called leukorrhea and is 2. Hydralazine (Apresoline)
caused by increased: 3. Narcan
1. Metabolic rates 4. RhoGAM
2. Production of estrogen 47. Which of the following answers best describes the
3. Functioning of the Bartholin glands stage of pregnancy in which maternal and fetal blood
4. Supply of sodium chloride to the cells of the vagina are exchanged?
37. A 26-year old multigravida is 14 weeks’ pregnant 1. Conception
and is scheduled for an alpha-fetoprotein test. She asks 2. 9 weeks’ gestation, when the fetal heart is well developed
the nurse, “What does the alpha-fetoprotein test 3. 32-34 weeks gestation
indicate?” The nurse bases a response on the 4. maternal and fetal blood are never exchanged
knowledge that this test can detect: 48. Gravida refers to which of the following
1. Kidney defects descriptions?
2. Cardiac defects 1. A serious pregnancy
3. Neural tube defects 2. Number of times a female has been pregnant
4. Urinary tract defects 3. Number of children a female has delivered
38. At a prenatal visit at 36 weeks’ gestation, a client 4. Number of term pregnancies a female has had.
complains of discomfort with irregularly occurring 49. A pregnant woman at 32 weeks’ gestation
contractions. The nurse instructs the client to: complains of feeling dizzy and lightheaded while her
1. Lie down until they stop fundal height is being measured. Her skin is pale and
2. Walk around until they subside moist. The nurse’s initial response would be to:
3. Time contraction for 30 minutes 1. Assess the woman’s blood pressure and pulse
4. Take 10 grains of aspirin for the discomfort 2. Have the woman breathe into a paper bag
39. The nurse teaches a pregnant woman to avoid lying 3. Raise the woman’s legs
on her back. The nurse has based this statement on the 4. Turn the woman on her side.
knowledge that the supine position can: 50. A pregnant woman’s last menstrual period began
1. Unduly prolong labor on April 8, 2005, and ended on April 13. Using
2. Cause decreased placental perfusion Naegele’s rule her estimated date of birth would be:
3. Lead to transient episodes of hypotension 1. January 15, 2006
4. Interfere with free movement of the coccyx 2. January 20, 2006
40. The pituitary hormone that stimulates the secretion 3. July 1, 2006
of milk from the mammary glands is: 4. November 5, 2005
1. Prolactin
2. Oxytocin Answers and Rationale
3. Estrogen 1. Answer: 1. FSH and LH are released from the
4. Progesterone anterior pituitary gland. FSH and LH, when stimulated by
41. Which of the following symptoms occurs with a gonadotropin-releasing hormone from the hypothalamus, are
hydatidiform mole? released from the anterior pituitary gland to stimulate
1. Heavy, bright red bleeding every 21 days follicular growth and development, growth of the graafian
2. Fetal cardiac motion after 6 weeks gestation follicle, and production of progesterone.
3. Benign tumors found in the smooth muscle of the uterus 2. Answer: 2. Two umbilical arteries and one umbilical
4. “Snowstorm” pattern on ultrasound with no fetus or vein. Blood pumped by the embryo’s heart leaves the embryo
gestational sac through two umbilical arteries. Once oxygenated, the blood
42. Which of the following terms applies to the then is returned by one umbilical vein. Arteries carry
tiny, blanched, slightly raised end arterioles found on deoxygenated blood and waste products from the fetus, and
the face, neck, arms, and chest during pregnancy? veins carry oxygenated blood and provide oxygen and
1. Epulis nutrients to the fetus.
2. Linea nigra 3. Answer: 3. 150 BPM. The fetal heart rate depends in
3. Striae gravidarum gestational age and ranges from 160-170 BPM in the first
4. Telangiectasias trimester but slows with fetal growth to 120-160 BPM near or
43. Which of the following conditions is common in at term. At or near term, if the fetal heart rate is less than
pregnant women in the 2nd trimester of pregnancy?
120 or more than 160 BPM with the uterus at rest, the fetus 12. Answer: 4. Wash the breasts with warm water and
may be in distress. keep them dry. The pregnant woman should be instructed to
4. Answer: 3. June 26, 2014. Accurate use of Naegele’s wash the breasts with warm water and keep them dry. The
rule requires that the woman have a regular 28-day woman should be instructed to avoid using soap on the
menstrual cycle. Add 7 days to the first day of the last nipples and areola area to prevent the drying of tissues.
menstrual period, subtract three months, and then add one Wearing a supportive bra with wide adjustable straps can
year to that date. decrease breast tenderness. Tight-fitting blouses or dresses
5. Answer: 2. G = 2, T = 0, P = 1, A = 0, L =1. Pregnancy will cause discomfort.
outcomes can be described with the acronym GTPAL. 13. Answer: 1. Any bleeding, such as in the gums,
“G” is Gravidity, the number of pregnancies. petechiae, and purpura. Severe Preeclampsia can trigger
“T” is term births, the number of born at term (38 to 41 disseminated intravascular coagulation because of the
weeks). widespread damage to vascular integrity. Bleeding is an early
“P” is preterm births, the number born before 38 weeks sign of DIC and should be reported to the M.D.
gestation. 14. Answer: 1. “I will maintain strict bedrest
“A” is abortions or miscarriages, included in “G” if before 20 throughout the remainder of pregnancy.” Strict bed rest
weeks gestation, included in parity if past 20 weeks AOE. throughout the remainder of pregnancy is not required. The
“L” is live births, the number of births of living children. woman is advised to curtail sexual activities until the bleeding
Therefore, a woman who is pregnant with twins and has a has ceased, and for 2 weeks following the last evidence of
child has a gravida of 2. Because the child was delivered at bleeding or as recommended by the physician. The woman is
37 weeks, the number of preterm births is 1, and the number instructed to count the number of perineal pads used daily
of term births is 0. The number of abortions is 0, and the and to note the quantity and color of blood on the pad. The
number of live births is 1. woman also should watch for the evidence of the passage of
6. Answer: 2. Fetal heart rate of 180 BPM. The normal tissue.
range of the fetal heart rate depends on gestational age. The 15. Answer: 3. “I need to drink unpasteurized milk
heart rate is usually 160-170 BPM in the first trimester and only.” All pregnant women should be advised to do the
slows with fetal growth, near and at term, the fetal heart rate following to prevent the development of toxoplasmosis.
ranges from 120-160 BPM. The other options are expected. Women should be instructed to cook meats thoroughly, avoid
7. Answer: 1. A softening of the cervix. In the early touching mucous membranes and eyes while handling raw
weeks of pregnancy the cervix becomes softer as a result of meat; thoroughly wash all kitchen surfaces that come into
increased vascularity and hyperplasia, which causes the contact with uncooked meat, wash the hands thoroughly after
Goodell’s sign. handling raw meat; avoid uncooked eggs and unpasteurized
8. Answer: 3. “It is the fetal movement that is felt by milk; wash fruits and vegetables before consumption, and
the mother.” Quickening is fetal movement and may occur avoid contact with materials that possibly are contaminated
as early as the 16th and 18th week of gestation, and the with cat feces, such as cat litter boxes, sandboxes, and
mother first notices subtle fetal movements that gradually garden soil.
increase in intensity. Braxton Hicks contractions are irregular, 16. Answer: 3. The client complains of a headache and
painless contractions that may occur throughout the blurred vision. If the client complains of a headache and
pregnancy. A thinning of the lower uterine segment occurs blurred vision, the physician should be notified because these
about the 6th week of pregnancy and is called Hegar’s sign. are signs of worsening Preeclampsia.
9. Answer: 4. Initiating a gentle upward tap on the 17. Answer: 3. “I need to avoid exercise because of the
cervix. Ballottement is a technique of palpating a floating negative effects of insulin production.” Exercise is safe
structure by bouncing it gently and feeling it rebound. In the for the client with gestational diabetes and is helpful in
technique used to palpate the fetus, the examiner places a lowering the blood glucose level.
finger in the vagina and taps gently upward, causing the fetus 18. Answer: 3. Respiratory rate of 10 BPM. Magnesium
to rise. The fetus then sinks, and the examiner feels a gentle sulfate depresses the respiratory rate. If the respiratory rate
tap on the finger. is less than 12 breaths per minute, the physician or other
10. Answers: 1, 4, 5, and 6. health care provider needs to be notified, and continuation of
The probable signs of pregnancy include: the medication needs to be reassessed. A urinary output of 20
Uterine Enlargement ml in a 30 minute period is adequate; less than 30 ml in one
Hegar’s sign or softening and thinning of the uterine segment hour needs to be reported. Deep tendon reflexes of 2+ are
that occurs at week 6. normal. The fetal heart rate is WNL for a resting fetus.
Goodell’s sign or softening of the cervix that occurs at the 19. Answer: 3. Clean and maintain an open airway. The
beginning of the 2nd month immediate care during a seizure (eclampsia) is to ensure a
Chadwick’s sign or bluish coloration of the mucous patent airway. The other options are actions that follow or will
membranes of the cervix, vagina and vulva. Occurs at week be implemented after the seizure has ceased.
6. 20. Answers: 1 Elevated blood pressure and 3 Facial
Ballottement or rebounding of the fetus against the edema. The three classic signs of preeclampsia are
examiner’s fingers of palpation hypertension, generalized edema, and proteinuria. Increased
Braxton-Hicks contractions respirations are not a sign of preeclampsia.
Positive pregnancy test measuring for hCG. 21. Answer: 1. Being affected by Rh incompatibility. Rh
Positive signs of pregnancy include: incompatibility can occur when an Rh-negative mom becomes
Fetal Heart Rate detected by electronic device (doppler) at sensitized to the Rh antigen. Sensitization may develop when
10-12 weeks an Rh-negative woman becomes pregnant with a fetus who is
Fetal Heart rate detected by nonelectronic device (fetoscope) Rh positive. During pregnancy and at delivery, some of the
at 20 weeks AOG baby’s Rh positive blood can enter the maternal circulation,
Active fetal movement palpable by the examiners causing the woman’s immune system to form antibodies
Outline of the fetus via radiography or ultrasound against Rh positive blood. Administration of Rho(D) immune
11. Answer: 1. Dorsiflex the foot while extending the globulin prevents the woman from developing antibodies
knee when the cramps occur. Legs cramps occur when the against Rh positive blood by providing passive antibody
pregnant woman stretches the leg and plantar flexes the foot. protection against the Rh antigen.
Dorsiflexion of the foot while extending the knee stretches the 22. Answer: 4. Respirations of 10 per
affected muscle, prevents the muscle from contracting, and minute. Magnesium toxicity can occur from magnesium
stops the cramping. sulfate therapy. Signs of toxicity relate to the central nervous
system depressant effects of the medication and include
respiratory depression, loss of deep tendon reflexes, and a the gravid uterus, which causes hypotension and decreased
sudden drop in the fetal heart rate and maternal heart rate systemic perfusion.
and blood pressure. Therapeutic levels of magnesium are 4-7 40. Answer: 1. Prolactin. Prolactin is the hormone from the
mEq/L. Proteinuria of +3 would be noted in a client with anterior pituitary gland that stimulates mammary gland
preeclampsia. secretion. Oxytocin, a posterior pituitary hormone, stimulates
23. Answer: 3. Seizures do not occur. For a client with the uterine musculature to contract and causes the “let down”
preeclampsia, the goal of care is directed at preventing reflex.
eclampsia (seizures). Magnesium sulfate is an anticonvulsant, 41. Answer: 4. “Snowstorm” pattern on ultrasound
not an antihypertensive agent. Although a decrease in blood with no fetus or gestational sac. The chorionic villi of a
pressure may be noted initially, this effect is usually transient. molar pregnancy resemble a snowstorm pattern on
Ankle clonus indicated hyperreflexia and may precede the ultrasound. Bleeding with a hydatidiform mole is often dark
onset of eclampsia. Scotomas are areas of complete or partial brown and may occur erratically for weeks or months.
blindness. Visual disturbances, such as scotomas, often 42. Answer: 4. Telangiectasias. The dilated arterioles that
precede an eclamptic seizure. occur during pregnancy are due to the elevated level of
24. Answers: 3, 4, 5, 6, and 7. When caring for a client circulating estrogen. The linea nigra is a pigmented line
receiving magnesium sulfate therapy, the nurse would extending from the symphysis pubis to the top of the fundus
monitor maternal vital signs, especially respirations, every during pregnancy.
30-60 minutes and notify the physician if respirations are less 43. Answer: 3. Physiologic anemia. Hemoglobin and
than 12, because this would indicate respiratory depression. hematocrit levels decrease during pregnancy as the increase
Calcium gluconate is kept on hand in case of magnesium in plasma volume exceeds the increase in red blood cell
sulfate overdose, because calcium gluconate is the antidote production.
for magnesium sulfate toxicity. Deep tendon reflexes are 44. Answer: 2. Electrolyte imbalance. Excessive vomiting
assessed hourly. Cardiac and renal function is monitored in clients with hyperemesis gravidarum often causes weight
closely. The urine output should be maintained at 30 ml per loss and fluid, electrolyte, and acid-base imbalances.
hour because the medication is eliminated through the 45. Answer: 1. Diet. Clients with gestational diabetes are
kidneys. usually managed by diet alone to control their glucose
25. Answer: 1. Administer RhoGAM within 72 intolerance. Oral hypoglycemic agents are contraindicated in
hours. RhoGAM is given within 72 hours postpartum if the pregnancy. NPH isn’t usually needed for blood glucose control
client has not been sensitized already. for GDM.
26. Answer: 2. Blood level of LH is too high. It is the 46. Answer: 1. Calcium gluconate. Calcium gluconate is
surge of LH secretion in mid cycle that is responsible for the antidote for magnesium toxicity. Ten ml of 10% calcium
ovulation. gluconate is given IV push over 3-5 minutes. Hydralazine is
27. Answer: 3. Preparation of the uterus to receive a given for sustained elevated blood pressures in preeclamptic
fertilized egg. Progesterone stimulates differentiation of the clients.
endometrium into a secretory type of tissue. 47. Answer: 4. maternal and fetal blood are never
28. Answer: 2. Eighth week to the time of birth. In the exchanged. Only nutrients and waste products are
first 7-14 days the ovum is known as a blastocyst; it is called transferred across the placenta. Blood exchange only occurs
an embryo until the eighth week; the developing cells are in complications and some medical procedures accidentally.
then called a fetus until birth. 48. Answer: 2. Number of times a female has been
29. Answer: 1. Placenta. When placental formation is pregnant. Gravida refers to the number of times a female
complete, around the 16th week of pregnancy; it produces has been pregnant, regardless of pregnancy outcome or the
estrogen and progesterone. number of neonates delivered.
30. Answer: 3. An increase in blood volume. The blood 49. Answer: 4. Turn the woman on her side. During a
volume increases by approximately 40-50% during fundal height measurement the woman is placed in a supine
pregnancy. The peak blood volume occurs between 30 and 34 position. This woman is experiencing supine hypotension as a
weeks of gestation. The hematocrit decreases as a result of result of uterine compression of the vena cava and abdominal
the increased blood volume. aorta. Turning her on her side will remove the compression
31. Answer: 4. Chadwick’s sign. A purplish color results and restore cardiac output and blood pressure. Then vital
from the increased vascularity and blood vessel engorgement signs can be assessed. Raising her legs will not solve the
of the vagina. problem since pressure will still remain on the major
32. Answer: 3. G5 T2 P1 A1 L4. 5 pregnancies; 2 term abdominal blood vessels, thereby continuing to impede
births; twins count as 1; one abortion; 4 living children. cardiac output. Breathing into a paper bag is the solution for
33. Answer: 4. Shortness of breath on exertion. This is dizziness related to respiratory alkalosis associated with
an expected cardiopulmonary adaptation during pregnancy; it hyperventilation.
is caused by an increased ventricular rate and elevated 50. Answer: 1. January 15, 2006. Naegele’s rule requires
diaphragm. subtracting 3 months and adding 7 days and 1 year if
34. Answer: 2. An increase of 300 calories a day. This is appropriate to the first day of a pregnant woman’s last
the recommended caloric increase for adult women to meet menstrual period. When this rule, is used with April 8, 2005,
the increased metabolic demands of pregnancy. the estimated date of birth is January 15, 2006.
35. Answer: 1. Acute hemolytic disease. When an Rh
negative mother carries an Rh positive fetus there is a risk for NCLEX Exam: Obstetrical Nursing – Intrapartum (60
maternal antibodies against Rh positive blood; antibodies Items)
cross the placenta and destroy the fetal RBC’s.
36. Answer: 2. Production of estrogen. The increase of 1. A nurse is caring for a client in labor. The nurse
estrogen during pregnancy causes hyperplasia of the vaginal determines that the client is beginning in the 2nd stage
mucosa, which leads to increased production of mucus by the of labor when which of the following assessments is
endocervical glands. The mucus contains exfoliated epithelial noted?
cells. 1. The client begins to expel clear vaginal fluid
37. Answer: 3. Neural tube defects. The alpha-fetoprotein 2. The contractions are regular
test detects neural tube defects and Down syndrome. 3. The membranes have ruptured
38. Answer: 2. Walk around until they 4. The cervix is dilated completely
subside. Ambulation relieves Braxton Hicks. 2. A nurse in the labor room is caring for a client in the
39. Answer: 2. Cause decreased placental active phases of labor. The nurse is assessing the fetal
perfusion. This is because impedance of venous return by
patterns and notes a late deceleration on the monitor 10. A nurse is admitting a pregnant client to the labor
strip. The most appropriate nursing action is to: room and attaches an external electronic fetal monitor
1. Place the mother in the supine position to the client’s abdomen. After attachment of the
2. Document the findings and continue to monitor the fetal monitor, the initial nursing assessment is which of the
patterns following?
3. Administer oxygen via face mask 1. Identifying the types of accelerations
4. Increase the rate of pitocin IV infusion 2. Assessing the baseline fetal heart rate
3. A nurse is performing an assessment of a client who 3. Determining the frequency of the contractions
is scheduled for a cesarean delivery. Which assessment 4. Determining the intensity of the contractions
finding would indicate a need to contact the physician? 11. A nurse is reviewing the record of a client in the
1. Fetal heart rate of 180 beats per minute labor room and notes that the nurse midwife has
2. White blood cell count of 12,000 documented that the fetus is at (-1) station. The nurse
3. Maternal pulse rate of 85 beats per minute determines that the fetal presenting part is:
4. Hemoglobin of 11.0 g/dL 1. 1 cm above the ischial spine
4. A client in labor is transported to the delivery room 2. 1 fingerbreadth below the symphysis pubis
and is prepared for a cesarean delivery. The client is 3. 1 inch below the coccyx
transferred to the delivery room table, and the nurse 4. 1 inch below the iliac crest
places the client in the: 12. A pregnant client is admitted to the labor room. An
1. Trendelenburg’s position with the legs in stirrups assessment is performed, and the nurse notes that the
2. Semi-Fowler position with a pillow under the knees client’s hemoglobin and hematocrit levels are low,
3. Prone position with the legs separated and elevated indicating anemia. The nurse determines that the client
4. Supine position with a wedge under the right hip is at risk for which of the following?
5. A nurse is caring for a client in labor and prepares to 1. A loud mouth
auscultate the fetal heart rate by using a Doppler 2. Low self-esteem
ultrasound device. The nurse most accurately 3. Hemorrhage
determines that the fetal heart sounds are heard by: 4. Postpartum infections
1. Noting if the heart rate is greater than 140 BPM 13. A nurse assists in the vaginal delivery of a newborn
2. Placing the diaphragm of the Doppler on the mother infant. After the delivery, the nurse observes the
abdomen umbilical cord lengthen and a spurt of blood from the
3. Performing Leopold’s maneuvers first to determine the vagina. The nurse documents these observations as
location of the fetal heart signs of:
4. Palpating the maternal radial pulse while listening to the 1. Hematoma
fetal heart rate 2. Placenta previa
6. A nurse is caring for a client in labor who is receiving 3. Uterine atony
Pitocin by IV infusion to stimulate uterine contractions. 4. Placental separation
Which assessment finding would indicate to the nurse 14. A client arrives at a birthing center in active labor.
that the infusion needs to be discontinued? Her membranes are still intact, and the nurse-midwife
1. Three contractions occurring within a 10-minute period prepares to perform an amniotomy. A nurse who is
2. A fetal heart rate of 90 beats per minute assisting the nurse-midwife explains to the client that
3. Adequate resting tone of the uterus palpated between after this procedure, she will most likely have:
contractions 1. Less pressure on her cervix
4. Increased urinary output 2. Increased efficiency of contractions
7. A nurse is beginning to care for a client in labor. The 3. Decreased number of contractions
physician has prescribed an IV infusion of Pitocin. The 4. The need for increased maternal blood pressure monitoring
nurse ensures that which of the following is 15. A nurse is monitoring a client in labor. The nurse
implemented before initiating the infusion? suspects umbilical cord compression if which of the
1. Placing the client on complete bed rest following is noted on the external monitor tracing
2. Continuous electronic fetal monitoring during a contraction?
3. An IV infusion of antibiotics 1. Early decelerations
4. Placing a code cart at the client’s bedside 2. Variable decelerations
8. A nurse is monitoring a client in active labor and 3. Late decelerations
notes that the client is having contractions every 3 4. Short-term variability
minutes that last 45 seconds. The nurse notes that the 16. A nurse explains the purpose of effleurage to a
fetal heart rate between contractions is 100 BPM. client in early labor. The nurse tells the client that
Which of the following nursing actions is most effleurage is:
appropriate? 1. A form of biofeedback to enhance bearing down efforts
1. Encourage the client’s coach to continue to encourage during delivery
breathing exercises 2. Light stroking of the abdomen to facilitate relaxation during
2. Encourage the client to continue pushing with each labor and provide tactile stimulation to the fetus
contraction 3. The application of pressure to the sacrum to relieve a
3. Continue monitoring the fetal heart rate backache
4. Notify the physician or nurse midwife 4. Performed to stimulate uterine activity by contracting a
9. A nurse is caring for a client in labor and is specific muscle group while other parts of the body rest
monitoring the fetal heart rate patterns. The nurse 17. A nurse is caring for a client in the second stage of
notes the presence of episodic accelerations on the labor. The client is experiencing uterine contractions
electronic fetal monitor tracing. Which of the following every 2 minutes and cries out in pain with each
actions is most appropriate? contraction. The nurse recognizes this behavior as:
1. Document the findings and tell the mother that the monitor 1. Exhaustion
indicates fetal well-being 2. Fear of losing control
2. Take the mother’s vital signs and tell the mother that bed 3. Involuntary grunting
rest is required to conserve oxygen. 4. Valsalva’s maneuver
3. Notify the physician or nurse midwife of the findings. 18. A nurse is monitoring a client in labor who is
4. Reposition the mother and check the monitor for changes receiving Pitocin and notes that the client is
in the fetal tracing
experiencing hypertonic uterine contractions. List in 1. Place the client in Trendelenburg’s position
order of priority the actions that the nurse takes. 2. Call the delivery room to notify the staff that the client will
1. Stop of Pitocin infusion be transported immediately
2. Perform a vaginal examination 3. Gently push the cord into the vagina
3. Reposition the client 4. Find the closest telephone and stat page the physician
4. Check the client’s blood pressure and heart rate 26. A maternity nurse is caring for a client with
5. Administer oxygen by face mask at 8 to 10 L/min abruptio placenta and is monitoring the client for
19. A nurse is assigned to care for a client with disseminated intravascular coagulopathy. Which
hypotonic uterine dysfunction and signs of a slowing assessment finding is least likely to be associated with
labor. The nurse is reviewing the physician’s orders and disseminated intravascular coagulation?
would expect to note which of the following prescribed 1. Swelling of the calf in one leg
treatments for this condition? 2. Prolonged clotting times
1. Medication that will provide sedation 3. Decreased platelet count
2. Increased hydration 4. Petechiae, oozing from injection sites, and hematuria
3. Oxytocin (Pitocin) infusion 27. A nurse is assessing a pregnant client in the
4. Administration of a tocolytic medication 2nd trimester of pregnancy who was admitted to the
20. A nurse in the labor room is preparing to care for a maternity unit with a suspected diagnosis of abruptio
client with hypertonic uterine dysfunction. The nurse is placentae. Which of the following assessment findings
told that the client is experiencing uncoordinated would the nurse expect to note if this condition is
contractions that are erratic in their frequency, present?
duration, and intensity. The priority nursing 1. Absence of abdominal pain
intervention would be to: 2. A soft abdomen
1. Monitor the Pitocin infusion closely 3. Uterine tenderness/pain
2. Provide pain relief measures 4. Painless, bright red vaginal bleeding
3. Prepare the client for an amniotomy 28. A maternity nurse is preparing for the admission of
4. Promote ambulation every 30 minutes a client in the 3rd trimester of pregnancy that is
21. A nurse is developing a plan of care for a client experiencing vaginal bleeding and has a suspected
experiencing dystocia and includes several nursing diagnosis of placenta previa. The nurse reviews the
interventions in the plan of care. The nurse prioritizes physician’s orders and would question which order?
the plan of care and selects which of the following 1. Prepare the client for an ultrasound
nursing interventions as the highest priority? 2. Obtain equipment for external electronic fetal heart
1. Keeping the significant other informed of the progress of monitoring
the labor 3. Obtain equipment for a manual pelvic examination
2. Providing comfort measures 4. Prepare to draw a Hgb and Hct blood sample
3. Monitoring fetal heart rate 29. An ultrasound is performed on a client at term
4. Changing the client’s position frequently gestation that is experiencing moderate vaginal
22. A maternity nurse is preparing to care for a bleeding. The results of the ultrasound indicate that an
pregnant client in labor who will be delivering twins. abruptio placenta is present. Based on these findings,
The nurse monitors the fetal heart rates by placing the the nurse would prepare the client for:
external fetal monitor: 1. Complete bed rest for the remainder of the pregnancy
1. Over the fetus that is most anterior to the mother’s 2. Delivery of the fetus
abdomen 3. Strict monitoring of intake and output
2. Over the fetus that is most posterior to the mother’s 4. The need for weekly monitoring of coagulation studies until
abdomen the time of delivery
3. So that each fetal heart rate is monitored separately 30. A nurse in a labor room is assisting with the vaginal
4. So that one fetus is monitored for a 15-minute period delivery of a newborn infant. The nurse would monitor
followed by a 15 minute fetal monitoring period for the the client closely for the risk of uterine rupture if which
second fetus of the following occurred?
23. A nurse in the postpartum unit is caring for a client 1. Hypotonic contractions
who has just delivered a newborn infant following a 2. Forceps delivery
pregnancy with placenta previa. The nurse reviews the 3. Schultz delivery
plan of care and prepares to monitor the client for 4. Weak bearing down efforts
which of the following risks associated with placenta 31. A client is admitted to the birthing suite in early
previa? active labor. The priority nursing intervention on
1. Disseminated intravascular coagulation admission of this client would be:
2. Chronic hypertension 1. Auscultating the fetal heart
3. Infection 2. Taking an obstetric history
4. Hemorrhage 3. Asking the client when she last ate
24. A nurse in the delivery room is assisting with the 4. Ascertaining whether the membranes were ruptured
delivery of a newborn infant. After the delivery of the 32. A client who is gravida 1, para 0 is admitted in
newborn, the nurse assists in delivering the placenta. labor. Her cervix is 100% effaced, and she is dilated to
Which observation would indicate that the placenta has 3 cm. Her fetus is at +1 station. The nurse is aware
separated from the uterine wall and is ready for that the fetus’ head is:
delivery? 1. Not yet engaged
1. The umbilical cord shortens in length and changes in color 2. Entering the pelvic inlet
2. A soft and boggy uterus 3. Below the ischial spines
3. Maternal complaints of severe uterine cramping 4. Visible at the vaginal opening
4. Changes in the shape of the uterus 33. After doing Leopold’s maneuvers, the nurse
25. A nurse in the labor room is performing a vaginal determines that the fetus is in the ROP position. To
assessment on a pregnant client in labor. The nurse best auscultate the fetal heart tones, the Doppler is
notes the presence of the umbilical cord protruding placed:
from the vagina. Which of the following would be the 1. Above the umbilicus at the midline
initial nursing action? 2. Above the umbilicus on the left side
3. Below the umbilicus on the right side 43. A laboring client is to have a pudendal block. The
4. Below the umbilicus near the left groin nurse plans to tell the client that once the block is
34. The physician asks the nurse the frequency of a working she:
laboring client’s contractions. The nurse assesses the 1. Will not feel the episiotomy
client’s contractions by timing from the beginning of 2. May lose bladder sensation
one contraction: 3. May lose the ability to push
1. Until the time it is completely over 4. Will no longer feel contractions
2. To the end of a second contraction 44. Which of the following observations indicates fetal
3. To the beginning of the next contraction distress?
4. Until the time that the uterus becomes very firm 1. Fetal scalp pH of 7.14
35. The nurse observes the client’s amniotic fluid and 2. Fetal heart rate of 144 beats/minute
decides that it appears normal, because it is: 3. Acceleration of fetal heart rate with contractions
1. Clear and dark amber in color 4. Presence of long term variability
2. Milky, greenish yellow, containing shreds of mucus 45. Which of the following fetal positions is most
3. Clear, almost colorless, and containing little white specks favorable for birth?
4. Cloudy, greenish-yellow, and containing little white specks 1. Vertex presentation
36. At 38 weeks gestation, a client is having late 2. Transverse lie
decelerations. The fetal pulse oximeter shows 75% to 3. Frank breech presentation
85%. The nurse should: 4. Posterior position of the fetal head
1. Discontinue the catheter, if the reading is not above 80% 46. A laboring client has external electronic fetal
2. Discontinue the catheter, if the reading does not go below monitoring in place. Which of the following assessment
30% data can be determined by examining the fetal heart
3. Advance the catheter until the reading is above 90% and rate strip produced by the external electronic fetal
continue monitoring monitor?
4. Reposition the catheter, recheck the reading, and if it is 1. Gender of the fetus
55%, keep monitoring 2. Fetal position
37. When examining the fetal monitor strip after 3. Labor progress
rupture of the membranes in a laboring client, the 4. Oxygenation
nurse notes variable decelerations in the fetal heart 47. A laboring client is in the first stage of labor and
rate. The nurse should: has progressed from 4 to 7 cm in cervical dilation. In
1. Stop the oxytocin infusion which of the following phases of the first stage does
2. Change the client’s position cervical dilation occur most rapidly?
3. Prepare for immediate delivery 1. Preparatory phase
4. Take the client’s blood pressure 2. Latent phase
38. When monitoring the fetal heart rate of a client in 3. Active phase
labor, the nurse identifies an elevation of 15 beats 4. Transition phase
above the baseline rate of 135 beats per minute lasting 48. A multiparous client who has been in labor for 2
for 15 seconds. This should be documented as: hours states that she feels the urge to move her
1. An acceleration bowels. How should the nurse respond?
2. An early elevation 1. Let the client get up to use the potty
3. A sonographic motion 2. Allow the client to use a bedpan
4. A tachycardic heart rate 3. Perform a pelvic examination
39. A laboring client complains of low back pain. The 4. Check the fetal heart rate
nurse replies that this pain occurs most when the 49. Labor is a series of events affected by the
position of the fetus is: coordination of the five essential factors. One of these
1. Breech is the passenger (fetus). Which are the other four
2. Transverse factors?
3. Occiput anterior 1. Contractions, passageway, placental position and function,
4. Occiput posterior pattern of care
40. The breathing technique that the mother should be 2. Contractions, maternal response, placental position,
instructed to use as the fetus’ head is crowning is: psychological response
1. Blowing 3. Passageway, contractions, placental position and function,
2. Slow chest psychological response
3. Shallow 4. Passageway, placental position and function, paternal
4. Accelerated-decelerated response, psychological response
41. During the period of induction of labor, a client 50. Fetal presentation refers to which of the following
should be observed carefully for signs of: descriptions?
1. Severe pain 1. Fetal body part that enters the maternal pelvis first
2. Uterine tetany 2. Relationship of the presenting part to the maternal pelvis
3. Hypoglycemia 3. Relationship of the long axis of the fetus to the long axis of
4. Umbilical cord prolapse the mother
42. A client arrives at the hospital in the second stage 4. A classification according to the fetal part
of labor. The fetus’ head is crowning, the client is 51. A client is admitted to the L & D suite at 36 weeks’
bearing down, and the birth appears imminent. The gestation. She has a history of C-section and complains
nurse should: of severe abdominal pain that started less than 1 hour
1. Transfer her immediately by stretcher to the birthing unit earlier. When the nurse palpates tetanic contractions,
2. Tell her to breathe through her mouth and not to bear the client again complains of severe pain. After the
down client vomits, she states that the pain is better and
3. Instruct the client to pant during contractions and to then passes out. Which is the probable cause of her
breathe through her mouth signs and symptoms?
4. Support the perineum with the hand to prevent tearing and 1. Hysteria compounded by the flu
tell the client to pant 2. Placental abruption
3. Uterine rupture
4. Dysfunctional labor
52. Upon completion of a vaginal examination on a 1. Having the children choose or make a gift to give to the
laboring woman, the nurse records: 50%, 6 cm, -1. new baby upon its arrival home
Which of the following is a correct interpretation of the 2. Emphasizing activities that keep the new baby and other
data? children together
1. Fetal presenting part is 1 cm above the ischial spines 3. Having the mother carry the new baby into the home so
2. Effacement is 4 cm from completion she can show the other children the new baby
3. Dilation is 50% completed 4. Reducing stress on other children by limiting their
4. Fetus has achieved passage through the ischial spines involvement in the care of the new baby
53. Which of the following findings meets the criteria of
a reassuring FHR pattern? Answers and Rationale
1. FHR does not change as a result of fetal activity 1. Answer: 4. The cervix is dilated completely. The
2. Average baseline rate ranges between 100 – 140 BPM second stage of labor begins when the cervix is dilated
3. Mild late deceleration patterns occur with some completely and ends with the birth of the neonate.
contractions 2. Answer: 3. Administer oxygen via face mask. Late
4. Variability averages between 6 – 10 BPM decelerations are due to uteroplacental insufficiency as the
54. Late deceleration patterns are noted when result of decreased blood flow and oxygen to the fetus during
assessing the monitor tracing of a woman whose labor the uterine contractions. This causes hypoxemia; therefore
is being induced with an infusion of Pitocin. The oxygen is necessary. The supine position is avoided because it
woman is in a side-lying position, and her vital signs decreases uterine blood flow to the fetus. The client should be
are stable and fall within a normal range. Contractions turned to her side to displace pressure of the gravid uterus on
are intense, last 90 seconds, and occur every 1 1/2 to 2 the inferior vena cava. An intravenous pitocin infusion is
minutes. The nurse’s immediate action would be to: discontinued when a late deceleration is noted.
1. Change the woman’s position 3. Answer: 1. Fetal heart rate of 180 beats per
2. Stop the Pitocin minute. A normal fetal heart rate is 120-160 beats per
3. Elevate the woman’s legs minute. A count of 180 beats per minute could indicate fetal
4. Administer oxygen via a tight mask at 8 to 10 liters/minute distress and would warrant physician notification. By full
55. The nurse should realize that the most common and term, a normal maternal hemoglobin range is 11-13 g/dL as a
potentially harmful maternal complication of epidural result of the hemodilution caused by an increase in plasma
anesthesia would be: volume during pregnancy.
1. Severe postpartum headache 4. Answer: 4. Supine position with a wedge under the
2. Limited perception of bladder fullness right hip. Vena cava and descending aorta compression by
3. Increase in respiratory rate the pregnant uterus impedes blood return from the lower
4. Hypotension trunk and extremities. This leads to decreasing cardiac return,
56. Perineal care is an important infection control cardiac output, and blood flow to the uterus and the fetus.
measure. When evaluating a postpartum woman’s The best position to prevent this would be side-lying with the
perineal care technique, the nurse would recognize the uterus displaced off of abdominal vessels. Positioning for
need for further instruction if the woman: abdominal surgery necessitates a supine position; however, a
1. Uses soap and warm water to wash the vulva and wedge placed under the right hip provides displacement of
perineum the uterus.
2. Washes from symphysis pubis back to episiotomy 5. Answer: 4. Palpating the maternal radial pulse while
3. Changes her perineal pad every 2 – 3 hours listening to the fetal heart rate. The nurse simultaneously
4. Uses the peribottle to rinse upward into her vagina should palpate the maternal radial or carotid pulse and
57. Which measure would be least effective in auscultate the fetal heart rate to differentiate the two. If the
preventing postpartum hemorrhage? fetal and maternal heart rates are similar, the nurse may
1. Administer Methergine 0.2 mg every 6 hours for 4 doses as mistake the maternal heart rate for the fetal heart rate.
ordered Leopold’s maneuvers may help the examiner locate the
2. Encourage the woman to void every 2 hours position of the fetus but will not ensure a distinction between
3. Massage the fundus every hour for the first 24 hours the two rates.
following birth 6. Answer: 2. A fetal heart rate of 90 beats per
4. Teach the woman the importance of rest and nutrition to minute. A normal fetal heart rate is 120-160 BPM.
enhance healing Bradycardia or late or variable decelerations indicate fetal
58. When making a visit to the home of a postpartum distress and the need to discontinue to pitocin. The goal of
woman one week after birth, the nurse should labor augmentation is to achieve three good-quality
recognize that the woman would characteristically: contractions in a 10-minute period.
1. Express a strong need to review events and her behavior 7. Answer: 2. Continuous electronic fetal
during the process of labor and birth monitoring. Continuous electronic fetal monitoring should be
2. Exhibit a reduced attention span, limiting readiness to implemented during an IV infusion of Pitocin.
learn 8. Answer: 4. Notify the physician or nurse midwife. A
3. Vacillate between the desire to have her own nurturing normal fetal heart rate is 120-160 beats per minute. Fetal
needs met and the need to take charge of her own care and bradycardia between contractions may indicate the need for
that of her newborn immediate medical management, and the physician or nurse
4. Have reestablished her role as a spouse/partner midwife needs to be notified.
59. Four hours after a difficult labor and birth, a 9. Answer: 1. Document the findings and tell the
primiparous woman refuses to feed her baby, stating mother that the monitor indicates fetal well-
that she is too tired and just wants to sleep. The nurse being. Accelerations are transient increases in the fetal heart
should: rate that often accompany contractions or are caused by fetal
1. Tell the woman she can rest after she feeds her baby movement. Episodic accelerations are thought to be a sign of
2. Recognize this as a behavior of the taking-hold stage fetal-well being and adequate oxygen reserve.
3. Record the behavior as ineffective maternal-newborn 10. Answer: 2. Assessing the baseline fetal heart
attachment rate. Assessing the baseline fetal heart rate is important so
4. Take the baby back to the nursery, reassuring the woman that abnormal variations of the baseline rate will be identified
that her rest is a priority at this time if they occur. Options 1 and 3 are important to assess, but
60. Parents can facilitate the adjustment of their other not as the first priority.
children to a new baby by:
11. Answer: 1. 1 cm above the ischial spine. Station is oxygenation. The mother should be positioned with the hips
the relationship of the presenting part to an imaginary line higher than the head to shift the fetal presenting part toward
drawn between the ischial spines, is measured in centimeters, the diaphragm. The nurse should push the call light to
and is noted as a negative number above the line and a summon help, and other staff members should call the
positive number below the line. At -1 station, the fetal physician and notify the delivery room. No attempt should be
presenting part is 1 cm above the ischial spines. made to replace the cord. The examiner, however, may place
12. Answer: 4. Postpartum infections. Anemic women a gloved hand into the vagina and hold the presenting part off
have a greater likelihood of cardiac decompensation during of the umbilical cord. Oxygen at 8 to 10 L/min by face mask
labor, postpartum infection, and poor wound healing. Anemia is delivered to the mother to increase fetal oxygenation.
does not specifically present a risk for hemorrhage. 26. Answer: 1. Swelling of the calf in one leg. DIC is a
13. Answer: 4. Placental separation. As the placenta state of diffuse clotting in which clotting factors are
separates, it settles downward into the lower uterine consumed, leading to widespread bleeding. Platelets are
segment. The umbilical cord lengthens, and a sudden trickle decreased because they are consumed by the process;
or spurt of blood appears. coagulation studies show no clot formation (and are thus
14. Answer: 2. Increased efficiency of normal to prolonged); and fibrin plugs may clog the
contractions. Amniotomy can be used to induce labor when microvasculature diffusely, rather than in an isolated area.
the condition of the cervix is favorable (ripe) or to augment The presence of petechiae, oozing from injection sites, and
labor if the process begins to slow. Rupturing of membranes hematuria are signs associated with DIC. Swelling and pain in
allows the fetal head to contact the cervix more directly and the calf of one leg are more likely to be associated with
may increase the efficiency of contractions. thrombophlebitis.
15. Answer: 2. Variable decelerations. Variable 27. 3. Uterine tenderness/pain. In abruptio placentae,
decelerations occur if the umbilical cord becomes acute abdominal pain is present. Uterine tenderness and pain
compressed, thus reducing blood flow between the placenta accompanies placental abruption, especially with a central
and the fetus. Early decelerations result from pressure on the abruption and trapped blood behind the placenta. The
fetal head during a contraction. Late decelerations are an abdomen will feel hard and boardlike on palpation as the
ominous pattern in labor because it suggests uteroplacental blood penetrates the myometrium and causes uterine
insufficiency during a contraction. Short-term variability irritability. Observation of the fetal monitoring often reveals
refers to the beat-to-beat range in the fetal heart rate. increased uterine resting tone, caused by failure of the uterus
16. Answer: 2. Light stroking of the abdomen to to relax in attempt to constrict blood vessels and control
facilitate relaxation during labor and provide tactile bleeding.
stimulation to the fetus. Effleurage is a specific type of 28. Answer: 3. Obtain equipment for a manual pelvic
cutaneous stimulation involving light stroking of the abdomen examination. Manual pelvic examinations are
and is used before transition to promote relaxation and contraindicated when vaginal bleeding is apparent in the
relieve mild to moderate pain. Effleurage provides tactile 3rd trimester until a diagnosis is made and placental previa is
stimulation to the fetus. ruled out. Digital examination of the cervix can lead to
17. Answer: 2. Fear of losing control. Pains, maternal and fetal hemorrhage. A diagnosis of placenta
helplessness, panicking, and fear of losing control are possible previa is made by ultrasound. The H/H levels are monitored,
behaviors in the 2nd stage of labor. and external electronic fetal heart rate monitoring is initiated.
18. Answer: 1, 4, 2. 5, 3. If uterine hypertonicity occurs, External fetal monitoring is crucial in evaluating the fetus that
the nurse immediately would intervene to reduce uterine is at risk for severe hypoxia.
activity and increase fetal oxygenation. The nurse would stop 29. Answer: 2. Delivery of the fetus. The goal of
the Pitocin infusion and increase the rate of the nonadditive management in abruptio placentae is to control the
solution, check maternal BP for hyper or hypotension, position hemorrhage and deliver the fetus as soon as possible.
the woman in a side-lying position, and administer oxygen by Delivery is the treatment of choice if the fetus is at term
snug face mask at 8-10 L/min. The nurse then would attempt gestation or if the bleeding is moderate to severe and the
to determine the cause of the uterine hypertonicity and mother or fetus is in jeopardy.
perform a vaginal exam to check for prolapsed cord. 30. Answer: 2. Forceps delivery. Excessive fundal
19. Answer: 3. Oxytocin (Pitocin) infusion. Therapeutic pressure, forceps delivery, violent bearing down efforts,
management for hypotonic uterine dysfunction includes tumultuous labor, and shoulder dystocia can place a woman
oxytocin augmentation and amniotomy to stimulate a labor at risk for traumatic uterine rupture. Hypotonic contractions
that slows. and weak bearing down efforts do not alone add to the risk of
20. Answer: 2. Provide pain relief rupture because they do not add to the stress on the uterine
measures. Management of hypertonic labor depends on the wall.
cause. Relief of pain is the primary intervention to promote a 31. Answer: 1. Auscultating the fetal heart. Determining
normal labor pattern. the fetal well-being supersedes all other measures. If the FHR
21. Answer: 3. Monitoring fetal heart rate. The priority is is absent or persistently decelerating, immediate intervention
to monitor the fetal heart rate. is required.
22. Answer: 3. So that each fetal heart rate is 32. Answer: 3. Below the ischial spines. A station of +1
monitored separately. In a client with a multi-fetal indicates that the fetal head is 1 cm below the ischial spines.
pregnancy, each fetal heart rate is monitored separately. 33. Answer: 3. Below the umbilicus on the right
23. Answer: 4. Hemorrhage. Because the placenta is side. Fetal heart tones are best auscultated through the fetal
implanted in the lower uterine segment, which does not back; because the position is ROP (right occiput presenting),
contain the same intertwining musculature as the fundus of the back would be below the umbilicus and on the right side.
the uterus, this site is more prone to bleeding. 34. Answer: 3. To the beginning of the next
24. Answer: 4. Changes in the shape of the contraction. This is the way to determine the frequency of
uterus. Signs of placental separation include lengthening of the contractions
the umbilical cord, a sudden gush of dark blood from the 35. Answer: 3. Clear, almost colorless, and containing
introitus (vagina), a firmly contracted uterus, and the uterus little white specks. By 36 weeks’ gestation, normal
changing from a discoid (like a disk) to a globular (like a amniotic fluid is colorless with small particles of vernix
globe) shape. The client may experience vaginal fullness, but caseosa present.
not severe uterine cramping. 36. Answer: 4. Reposition the catheter, recheck the
25. Answer: 1. Place the client in Trendelenburg’s reading, and if it is 55%, keep monitoring. Adjusting the
position. When cord prolapse occurs, prompt actions are catheter would be indicated. Normal fetal pulse oximetry
taken to relieve cord compression and increase fetal
should be between 30% and 70%. 75% to 85% would fetal lie; the three possible lies are longitudinal, transverse,
indicate maternal readings. and oblique.
37. Answer: 2. Change the client’s position. Variable 51. Answer: 3. Uterine rupture. Uterine rupture is a
decelerations usually are seen as a result of cord medical emergency that may occur before or during labor.
compression; a change of position will relieve pressure on the Signs and symptoms typically include abdominal pain that
cord. may ease after uterine rupture, vomiting, vaginal bleeding,
38. Answer: 1. An acceleration. An acceleration is an hypovolemic shock, and fetal distress. With placental
abrupt elevation above the baseline of 15 beats per minute abruption, the client typically complains of vaginal bleeding
for 15 seconds; if the acceleration persists for more than 10 and constant abdominal pain.
minutes it is considered a change in baseline rate. A 52. Answer: 1. Fetal presenting part is 1 cm above the
tachycardic FHR is above 160 beats per minute. ischial spines. Station of – 1 indicates that the fetal
39. Answer: 4. Occiput posterior. A persistent occiput- presenting part is above the ischial spines and has not yet
posterior position causes intense back pain because of fetal passed through the pelvic inlet. A station of zero would
compression of the sacral nerves. Occiput anterior is the most indicate that the presenting part has passed through the inlet
common fetal position and does not cause back pain. and is at the level of the ischial spines or is
40. Answer: 1. Blowing. Blowing forcefully through the engaged. Passage through the ischial spines with internal
mouth controls the strong urge to push and allows for a more rotation would be indicated by a plus station, such as +
controlled birth of the head. 1. Progress of effacement is referred to by percentages with
41. Answer: 2. Uterine tetany. Uterine tetany could result 100% indicating full effacement and dilation by centimeters
from the use of oxytocin to induce labor. Because oxytocin (cm) with 10 cm indicating full dilation.
promotes powerful uterine contractions, uterine tetany may 53. Answer: 4. Variability averages between 6 – 10
occur. The oxytocin infusion must be stopped to prevent BPM. Variability indicates a well oxygenated fetus with a
uterine rupture and fetal compromise. functioning autonomic nervous system. FHR should accelerate
42. Answer: 4. Support the perineum with the hand to with fetal movement. Baseline range for the FHR is 120 to
prevent tearing and tell the client to pant. Gentle 160 beats per minute. Late deceleration patterns are never
pressure is applied to the baby’s head as it emerges so it is reassuring, though early and mild variable decelerations are
not born too rapidly. The head is never held back, and it expected, reassuring findings.
should be supported as it emerges so there will be no vaginal 54. Answer: 2. Stop the Pitocin. Late deceleration patterns
lacerations. It is impossible to push and pant at the same noted are most likely related to alteration in uteroplacental
time. perfusion associated with the strong contractions
43. Answer: 1. May lose the ability to push. A pudendal described. The immediate action would be to stop the Pitocin
block provides anesthesia to the perineum. infusion since Pitocin is an oxytocic which stimulates the
44. Answer: 1. Fetal scalp pH of 7.14. A fetal scalp pH uterus to contract. The woman is already in an appropriate
below 7.25 indicates acidosis and fetal hypoxia. position for uteroplacental perfusion. Elevation of her legs
45. Answer: 1. Vertex presentation. Vertex presentation would be appropriate if hypotension were present. Oxygen is
(flexion of the fetal head) is the optimal presentation for appropriate but not the immediate action.
passage through the birth canal. Transverse lie is an 55. Answer: 4. Hypotension. Epidural anesthesia can lead
unacceptable fetal position for vaginal birth and requires a C- to vasodilation and a drop in blood pressure that could
section. Frank breech presentation, in which the buttocks interfere with adequate placental perfusion. The woman must
present first, can be a difficult vaginal delivery. Posterior be well hydrated before and during epidural anesthesia to
positioning of the fetal head can make it difficult for the fetal prevent this problem and maintain an adequate blood
head to pass under the maternal symphysis pubis. pressure. Headache is not a side effect since the spinal fluid
46. Answer: 4. Oxygenation. Oxygenation of the fetus may is not disturbed by this anesthetic as it would be with a low
be indirectly assessed through fetal monitoring by closely spinal (saddle block) anesthesia; 2 is an effect of epidural
examining the fetal heart rate strip. Accelerations in the fetal anesthesia but is not the most harmful. Respiratory
heart rate strip indicate good oxygenation, while depression is a potentially serious complication.
decelerations in the fetal heart rate sometimes indicate poor 56. Answer: 4. Uses the peribottle to rinse upward into
fetal oxygenation. her vagina. Responses 1, 2, and 3 are all appropriate
47. Answer: 3. Active phase. Cervical dilation occurs more measures. The peri bottle should be used in a backward
rapidly during the active phase than any of the previous direction over the perineum. The flow should never be
phases. The active phase is characterized by cervical dilation directed upward into the vagina since debris would be forced
that progresses from 4 to 7 cm. The preparatory, or latent, upward into the uterus through the still-open cervix.
phase begins with the onset of regular uterine contractions 57. Answer: 3. Massage the fundus every hour for the
and ends when rapid cervical dilation begins. Transition is first 24 hours following birth. The fundus should be
defined as cervical dilation beginning at 8 cm and lasting until massaged only when boggy or soft. Massaging a firm fundus
10 cm or complete dilation. could cause it to relax. Responses 1, 2, and 4 are all effective
48. Answer: 3. Perform a pelvic examination. A measures to enhance and maintain contraction of the uterus
complaint of rectal pressure usually indicates a low presenting and to facilitate healing.
fetal part, signaling imminent delivery. The nurse should 58. Answer: 3. Vacillate between the desire to have her
perform a pelvic examination to assess the dilation of the own nurturing needs met and the need to take charge
cervix and station of the presenting fetal part. of her own care and that of her newborn. One week after
49. Answer: 3. Passageway, contractions, placental birth the woman should exhibit behaviors characteristic of the
position and function, psychological response. The five taking-hold stage as described in response 3. This stage lasts
essential factors (5 P’s) are passenger (fetus), passageway for as long as 4 to 5 weeks after birth. Responses 1 and 2 are
(pelvis), powers (contractions), placental position and characteristic of the taking-in stage, which lasts for the first
function, and psyche (psychological response of the mother). few days after birth. Response 4 reflects the letting-go stage,
50. Answer: 1. Fetal body part that enters the maternal which indicates that psychosocial recovery is complete.
pelvis first. Presentation is the fetal body part that enters 59. Answer: 4. Take the baby back to the nursery,
the pelvis first; it’s classified by the presenting part; the three reassuring the woman that her rest is a priority at this
main presentations are cephalic/occipital, breech, and time. Response 1 does not take into consideration the need
shoulder. The relationship of the presenting fetal part to the for the new mother to be nurtured and have her needs met
maternal pelvis refers to fetal position. The relationship of the during the taking-in stage. The behavior described is typical
long axis to the fetus to the long axis of the mother refers to of this stage and not a reflection of ineffective attachment
unless the behavior persists. Mothers need to reestablish 7. A nurse in a PP unit is instructing a mother regarding
their own well-being in order to effectively care for their baby. lochia and the amount of expected lochia drainage. The
60. Answer: 1. Having the children choose or make a nurse instructs the mother that the normal amount of
gift to give to the new baby upon its arrival lochia may vary but should never exceed the need for:
home. Special time should be set aside just for the other 1. One peripad per day
children without interruption from the newborn. Someone 2. Two peripads per day
other than the mother should carry the baby into the home so 3. Three peripads per day
she can give full attention to greeting her other 4. Eight peripads per day
children. Children should be actively involved in the care of 8. A PP nurse is providing instructions to a woman after
the baby according to their ability without overwhelming delivery of a healthy newborn infant. The nurse
them. instructs the mother that she should expect normal
bowel elimination to return:
NCLEX Exam: Obstetrical Nursing – Postpartum (55 1. One the day of the delivery
Items) 2. 3 days PP
3. 7 days PP
1. A postpartum nurse is preparing to care for a woman 4. within 2 weeks PP
who has just delivered a healthy newborn infant. In the 9. Select all of the physiological maternal changes that
immediate postpartum period the nurse plans to take occur during the PP period.
the woman’s vital signs: 1. Cervical involution ceases immediately
1. Every 30 minutes during the first hour and then every hour 2. Vaginal distention decreases slowly
for the next two hours. 3. Fundus begins to descend into the pelvis after 24 hours
2. Every 15 minutes during the first hour and then every 30 4. Cardiac output decreases with resultant tachycardia in the
minutes for the next two hours. first 24 hours
3. Every hour for the first 2 hours and then every 4 hours 5. Digestive processes slow immediately.
4. Every 5 minutes for the first 30 minutes and then every 10. A nurse is caring for a PP woman who has received
hour for the next 4 hours. epidural anesthesia and is monitoring the woman for
2. A postpartum nurse is taking the vital signs of a the presence of a vulva hematoma. Which of the
woman who delivered a healthy newborn infant 4 hours following assessment findings would best indicate the
ago. The nurse notes that the mother’s temperature is presence of a hematoma?
100.2*F. Which of the following actions would be most 1. Complaints of a tearing sensation
appropriate? 2. Complaints of intense pain
1. Retake the temperature in 15 minutes 3. Changes in vital signs
2. Notify the physician 4. Signs of heavy bruising
3. Document the findings 11. A nurse is developing a plan of care for a PP woman
4. Increase hydration by encouraging oral fluids with a small vulvar hematoma. The nurse includes
3. The nurse is assessing a client who is 6 hours PP which specific intervention in the plan during the first
after delivering a full-term healthy infant. The client 12 hours following the delivery of this client?
complains to the nurse of feelings of faintness and 1. Assess vital signs every 4 hours
dizziness. Which of the following nursing actions would 2. Inform health care provider of assessment findings
be most appropriate? 3. Measure fundal height every 4 hours
1. Obtain hemoglobin and hematocrit levels 4. Prepare an ice pack for application to the area.
2. Instruct the mother to request help when getting out of 12. A new mother received epidural anesthesia during
bed labor and had a forceps delivery after pushing 2 hours.
3. Elevate the mother’s legs At 6 hours PP, her systolic blood pressure has dropped
4. Inform the nursery room nurse to avoid bringing the 20 points, her diastolic BP has dropped 10 points, and
newborn infant to the mother until the feelings of her pulse is 120 beats per minute. The client is anxious
lightheadedness and dizziness have subsided. and restless. On further assessment, a vulvar
4. A nurse is preparing to perform a fundal assessment hematoma is verified. After notifying the health care
on a postpartum client. The initial nursing action in provider, the nurse immediately plans to:
performing this assessment is which of the following? 1. Monitor fundal height
1. Ask the client to turn on her side 2. Apply perineal pressure
2. Ask the client to lie flat on her back with the knees and 3. Prepare the client for surgery.
legs flat and straight. 4. Reassure the client
3. Ask the mother to urinate and empty her bladder 13. A nurse is monitoring a new mother in the PP
4. Massage the fundus gently before determining the level of period for signs of hemorrhage. Which of the following
the fundus. signs, if noted in the mother, would be an early sign of
5. The nurse is assessing the lochia on a 1 day PP excessive blood loss?
patient. The nurse notes that the lochia is red and has a 1. A temperature of 100.4*F
foul-smelling odor. The nurse determines that this 2. An increase in the pulse from 88 to 102 BPM
assessment finding is: 3. An increase in the respiratory rate from 18 to 22 breaths
1. Normal per minute
2. Indicates the presence of infection 4. A blood pressure change from 130/88 to 124/80 mm Hg
3. Indicates the need for increasing oral fluids 14. A nurse is preparing to assess the uterine fundus of
4. Indicates the need for increasing ambulation a client in the immediate postpartum period. When the
6. When performing a PP assessment on a client, the nurse locates the fundus, she notes that the uterus
nurse notes the presence of clots in the lochia. The feels soft and boggy. Which of the following nursing
nurse examines the clots and notes that they are larger interventions would be most appropriate initially?
than 1 cm. Which of the following nursing actions is 1. Massage the fundus until it is firm
most appropriate? 2. Elevate the mothers legs
1. Document the findings 3. Push on the uterus to assist in expressing clots
2. Notify the physician 4. Encourage the mother to void
3. Reassess the client in 2 hours 15. A PP nurse is assessing a mother who delivered a
4. Encourage increased intake of fluids. healthy newborn infant by C-section. The nurse is
assessing for signs and symptoms of superficial venous
thrombosis. Which of the following signs or symptoms 1. Peripheral vascular disease
would the nurse note if superficial venous thrombosis 2. Hypothyroidism
were present? 3. Hypotension
1. Paleness of the calf area 4. Type 1 diabetes
2. Enlarged, hardened veins 24. Which of the following factors might result in a
3. Coolness of the calf area decreased supply of breastmilk in a PP mother?
4. Palpable dorsalis pedis pulses 1. Supplemental feedings with formula
16. A nurse is providing instructions to a mother who 2. Maternal diet high in vitamin C
has been diagnosed with mastitis. Which of the 3. An alcoholic drink
following statements if made by the mother indicates a 4. Frequent feedings
need for further teaching? 25. Which of the following interventions would be
1. “I need to take antibiotics, and I should begin to feel better helpful to a breastfeeding mother who is experiencing
in 24-48 hours.” engorged breasts?
2. “I can use analgesics to assist in alleviating some of the 1. Applying ice
discomfort.” 2. Applying a breast binder
3. “I need to wear a supportive bra to relieve the discomfort.” 3. Teaching how to express her breasts in a warm shower
4. “I need to stop breastfeeding until this condition resolves.” 4. Administering bromocriptine (Parlodel)
17. A PP client is being treated for DVT. The nurse 26. On completing a fundal assessment, the nurse
understands that the client’s response to treatment will notes the fundus is situated on the client’s left
be evaluated by regularly assessing the client for: abdomen. Which of the following actions is
1. Dysuria, ecchymosis, and vertigo appropriate?
2. Epistaxis, hematuria, and dysuria 1. Ask the client to empty her bladder
3. Hematuria, ecchymosis, and epistaxis 2. Straight catheterize the client immediately
4. Hematuria, ecchymosis, and vertigo 3. Call the client’s health provider for direction
18. A nurse performs an assessment on a client who is 4. Straight catheterize the client for half of her uterine
4 hours PP. The nurse notes that the client has cool, volume
clammy skin and is restless and excessively thirsty. The 27. The nurse is about the give a Type 2 diabetic her
nurse prepares immediately to: insulin before breakfast on her first day postpartum.
1. Assess for hypovolemia and notify the health care provider Which of the following answers best describes insulin
2. Begin hourly pad counts and reassure the client requirements immediately postpartum?
3. Begin fundal massage and start oxygen by mask 1. Lower than during her pregnancy
4. Elevate the head of the bed and assess vital signs 2. Higher than during her pregnancy
19. A nurse is assessing a client in the 4th stage if 3. Lower than before she became pregnant
labor and notes that the fundus is firm but that 4. Higher than before she became pregnant
bleeding is excessive. The initial nursing action would 28. Which of the following findings would be expected
be which of the following? when assessing the postpartum client?
1. Massage the fundus 1. Fundus 1 cm above the umbilicus 1 hour postpartum
2. Place the mother in the Trendelenburg’s position 2. Fundus 1 cm above the umbilicus on postpartum day 3
3. Notify the physician 3. Fundus palpable in the abdomen at 2 weeks postpartum
4. Record the findings 4. Fundus slightly to the right; 2 cm above umbilicus on
20. A nurse is caring for a PP client with a diagnosis of postpartum day 2
DVT who is receiving a continuous intravenous infusion 29. A client is complaining of painful contractions,
of heparin sodium. Which of the following laboratory or afterpains, on postpartum day 2. Which of the
results will the nurse specifically review to determine if following conditions could increase the severity of
an effective and appropriate dose of the heparin is afterpains?
being delivered? 1. Bottle-feeding
1. Prothrombin time 2. Diabetes
2. International normalized ratio 3. Multiple gestation
3. Activated partial thromboplastin time 4. Primiparity
4. Platelet count 30. On which of the postpartum days can the client
21. A nurse is preparing a list of self-care instructions expect lochia serosa?
for a PP client who was diagnosed with mastitis. Select 1. Days 3 and 4 PP
all instructions that would be included on the list. 2. Days 3 to 10 PP
1. Take the prescribed antibiotics until the soreness subsides. 3. Days 10-14 PP
2. Wear supportive bra 4. Days 14 to 42 PP
3. Avoid decompression of the breasts by breastfeeding or 31. Which of the following behaviors characterizes the
breast pump PP mother in the taking inphase?
4. Rest during the acute phase 1. Passive and dependant
5. Continue to breastfeed if the breasts are not too sore. 2. Striving for independence and autonomy
22. Methergine or pitocin is prescribed for a woman to 3. Curious and interested in care of the baby
treat PP hemorrhage. Before administration of these 4. Exhibiting maximum readiness for new learning
medications, the priority nursing assessment is to 32. Which of the following complications may be
check the: indicated by continuous seepage of blood from the
1. Amount of lochia vagina of a PP client, when palpation of the uterus
2. Blood pressure reveals a firm uterus 1 cm below the umbilicus?
3. Deep tendon reflexes 1. Retained placental fragments
4. Uterine tone 2. Urinary tract infection
23. Methergine or pitocin are prescribed for a client 3. Cervical laceration
with PP hemorrhage. Before administering the 4. Uterine atony
medication(s), the nurse contacts the health provider 33. What type of milk is present in the breasts 7 to 10
who prescribed the medication(s) in which of the days PP?
following conditions is documented in the client’s 1. Colostrum
medical history? 2. Hind milk
3. Mature milk understands that this is indicated for this client
4. Transitional milk because:
34. Which of the following complications is most likely 1. She had a precipitate birth
responsible for a delayed postpartum hemorrhage? 2. This was an extramural birth
1. Cervical laceration 3. Retained placental fragments must be expelled
2. Clotting deficiency 4. Multigravidas are at increased risk for uterine atony.
3. Perineal laceration 44. As part of the postpartum assessment, the nurse
4. Uterine subinvolution examines the breasts of a primiparous breastfeeding
35. Before giving a PP client the rubella vaccine, which woman who is one day postpartum. An expected
of the following facts should the nurse include in client finding would be:
teaching? 1. Soft, non-tender; colostrum is present
1. The vaccine is safe in clients with egg allergies 2. Leakage of milk at let down
2. Breast-feeding isn’t compatible with the vaccine 3. Swollen, warm, and tender upon palpation
3. Transient arthralgia and rash are common adverse effects 4. A few blisters and a bruise on each areola
4. The client should avoid getting pregnant for 3 months after 45. Following the birth of her baby, a woman expresses
the vaccine because the vaccine has teratogenic effects concern about the weight she gained during pregnancy
36. Which of the following changes best described the and how quickly she can lose it now that the baby is
insulin needs of a client with type 1 diabetes who has born. The nurse, in describing the expected pattern of
just delivered an infant vaginally without weight loss, should begin by telling this woman that:
complications? 1. Return to pre pregnant weight is usually achieved by the
1. Increase end of the postpartum period
2. Decrease 2. Fluid loss from diuresis, diaphoresis, and bleeding accounts
3. Remain the same as before pregnancy for about a 3 pound weight loss
4. Remain the same as during pregnancy 3. The expected weight loss immediately after birth averages
37. Which of the following responses is most about 11 to 13 pounds
appropriate for a mother with diabetes who wants to 4. Lactation will inhibit weight loss since caloric intake must
breastfeed her infant but is concerned about the effects increase to support milk production
of breastfeeding on her health? 46. Which of the following findings would be a source
1. Mothers with diabetes who breastfeed have a hard time of concern if noted during the assessment of a woman
controlling their insulin needs who is 12 hours postpartum?
2. Mothers with diabetes shouldn’t breastfeed because of 1. Postural hypotension
potential complications 2. Temperature of 100.4°F
3. Mothers with diabetes shouldn’t breastfeed; insulin 3. Bradycardia — pulse rate of 55 BPM
requirements are doubled. 4. Pain in left calf with dorsiflexion of left foot
4. Mothers with diabetes may breastfeed; insulin 47. The nurse examines a woman one hour after
requirements may decrease from breastfeeding. birth. The woman’s fundus is boggy, midline, and 1 cm
38. On the first PP night, a client requests that her below the umbilicus. Her lochial flow is profuse, with
baby be sent back to the nursery so she can get some two plum-sized clots. The nurse’s initial action would
sleep. The client is most likely in which of the following be to:
phases? 1. Place her on a bedpan to empty her bladder
1. Depression phase 2. Massage her fundus
2. Letting-go phase 3. Call the physician
3. Taking-hold phase 4. Administer Methergine 0.2 mg IM which has been ordered
4. Taking-in phase prn
39. Which of the following physiological responses is 48. When performing a postpartum check, the nurse
considered normal in the early postpartum period? should:
1. Urinary urgency and dysuria 1. Assist the woman into a lateral position with upper leg
2. Rapid diuresis flexed forward to facilitate the examination of her perineum
3. Decrease in blood pressure 2. Assist the woman into a supine position with her arms
4. Increase motility of the GI system above her head and her legs extended for the examination of
40. During the 3rd PP day, which of the following her abdomen
observations about the client would the nurse be most 3. Instruct the woman to avoid urinating just before the
likely to make? examination since a full bladder will facilitate fundal palpation
1. The client appears interested in learning about neonatal 4. Wash hands and put on sterile gloves before beginning the
care check
2. The client talks a lot about her birth experience 49. Perineal care is an important infection control
3. The client sleeps whenever the neonate isn’t present measure. When evaluating a postpartum woman’s
4. The client requests help in choosing a name for the perineal care technique, the nurse would recognize the
neonate. need for further instruction if the woman:
41. Which of the following circumstances is most likely 1. Uses soap and warm water to wash the vulva and
to cause uterine atony and lead to PP hemorrhage? perineum
1. Hypertension 2. Washes from symphysis pubis back to episiotomy
2. Cervical and vaginal tears 3. Changes her perineal pad every 2 – 3 hours
3. Urine retention 4. Uses the peribottle to rinse upward into her vagina
4. Endometritis 50. Which measure would be least effective in
42. Which type of lochia should the nurse expect to find preventing postpartum hemorrhage?
in a client 2 days PP? 1. Administer Methergine 0.2 mg every 6 hours for 4 doses as
1. Foul-smelling ordered
2. Lochia serosa 2. Encourage the woman to void every 2 hours
3. Lochia alba 3. Massage the fundus every hour for the first 24 hours
4. Lochia rubra following birth
43. After expulsion of the placenta in a client who has 4. Teach the woman the importance of rest and nutrition to
six living children, an infusion of lactated ringer’s enhance healing
solution with 10 units of pitocin is ordered. The nurse
51. When making a visit to the home of a postpartum performing fundal assessment, the nurse asks the woman to
woman one week after birth, the nurse should lie flat on her back with the knees flexed. Massaging the
recognize that the woman would characteristically: fundus is not appropriate unless the fundus is boggy and soft,
1. Express a strong need to review events and her behavior and then it should be massaged gently until firm.
during the process of labor and birth 5. Answer: 2. Indicates the presence of
2. Exhibit a reduced attention span, limiting readiness to infection. Lochia, the discharge present after birth, is red for
learn the first 1 to 3 days and gradually decreases in amount.
3. Vacillate between the desire to have her own nurturing Normal lochia has a fleshy odor. Foul smelling or purulent
needs met and the need to take charge of her own care and lochia usually indicates infection, and these findings are not
that of her newborn normal. Encouraging the woman to drink fluids or increase
4. Have reestablished her role as a spouse/partner ambulation is not an accurate nursing intervention.
52. Four hours after a difficult labor and birth, a 6. Answer: 2. Notify the physician. Normally, one may
primiparous woman refuses to feed her baby, stating find a few small clots in the first 1 to 2 days after birth from
that she is too tired and just wants to sleep. The nurse pooling of blood in the vagina. Clots larger than 1 cm are
should: considered abnormal. The cause of these clots, such as
1. Tell the woman she can rest after she feeds her baby uterine atony or retained placental fragments, needs to be
2. Recognize this as a behavior of the taking-hold stage determined and treated to prevent further blood loss.
3. Record the behavior as ineffective maternal-newborn Although the findings would be documented, the most
attachment appropriate action is to notify the physician.
4. Take the baby back to the nursery, reassuring the woman 7. Answer: 4. Eight peripads per day. The normal amount
that her rest is a priority at this time of lochia may vary with the individual but should never
53. Parents can facilitate the adjustment of their other exceed 4 to 8 peripads per day. The average number of
children to a new baby by: peripads is 6 per day.
1. Having the children choose or make a gift to give to the 8. Answer: 2. 3 days PP. After birth, the nurse should
new baby upon its arrival home auscultate the woman’s abdomen in all four quadrants to
2. Emphasizing activities that keep the new baby and other determine the return of bowel sounds. Normal bowel
children together elimination usually returns 2 to 3 days PP. Surgery,
3. Having the mother carry the new baby into the home so anesthesia, and the use of narcotics and pain control agents
she can show the other children the new baby also contribute to the longer period of altered bowel function.
4. Reducing stress on other children by limiting their 9. Answer: 1 and 3. In the PP period, cervical healing
involvement in the care of the new baby occurs rapidly and cervical involution occurs.After 1
54. A primiparous woman is in the taking-in stage of week the muscle begins to regenerate and the cervix feels
psychosocial recovery and adjustment following firm and the external os is the width of a pencil. Although the
birth. The nurse, recognizing the needs of women vaginal mucosa heals and vaginal distention decreases, it
during this stage, should: takes the entire PP period for complete involution to occur
1. Foster an active role in the baby’s care and muscle tone is never restored to the pregravid state. The
2. Provide time for the mother to reflect on the events of and fundus begins to descent into the pelvic cavity after 24 hours,
her behavior during childbirth a process known as involution. Despite blood loss that occurs
3. Recognize the woman’s limited attention span by giving her during delivery of the baby, a transient increase in cardiac
written materials to read when she gets home rather than output occurs. The increase in cardiac output, which persists
doing a teaching session now about 48 hours after childbirth, is probably caused by an
4. Promote maternal independence by encouraging her to increase in stroke volume because Bradycardia is often noted
meet her own hygiene and comfort needs during the PP period. Soon after childbirth, digestion begins to
55. All of the following are important in the immediate begin to be active and the new mother is usually hungry
care of the premature neonate. Which nursing activity because of the energy expended during labor.
should have the greatest priority? 10. Answer: 3. Changes in vital signs. Because the
1. Instillation of antibiotic in the eyes woman has had epidural anesthesia and is anesthetized, she
2. Identification by bracelet and foot prints cannot feel pain, pressure, or a tearing sensation. Changes in
3. Placement in a warm environment vitals indicate hypovolemia in the anesthetized PP woman
4. Neurological assessment to determine gestational age with vulvar hematoma. Heavy bruising may be visualized, but
vital sign changes indicate hematoma caused by blood
Answers and Rationale collection in the perineal tissues.
1. Answer: 2. Every 15 minutes during the first hour 11. Answer: 4. Prepare an ice pack for application to
and then every 30 minutes for the next two hours. the area. Application of ice will reduce swelling caused by
2. Answer: 4. Increase hydration by encouraging oral hematoma formation in the vulvar area. The other options are
fluids. The mother’s temperature may be taken every 4 not interventions that are specific to the plan of care for a
hours while she is awake. Temperatures up to 100.4 (38 C) in client with a small vulvar hematoma.
the first 24 hours after birth are often related to the 12. Answer: 3. Prepare the client for surgery. The use of
dehydrating effects of labor. The most appropriate action is to an epidural, prolonged second stage labor and forceps
increase hydration by encouraging oral fluids, which should delivery are predisposing factors for hematoma formation,
bring the temperature to a normal reading. Although the and a collection of up to 500 ml of blood can occur in the
nurse would document the findings, the most appropriate vaginal area. Although the other options may be
action would be to increase the hydration. implemented, the immediate action would be to prepare the
3. Answer: 2. Instruct the mother to request help when client for surgery to stop the bleeding.
getting out of bed. Orthostatic hypotension may be evident 13. Answer: 2. An increase in the pulse from 88 to 102
during the first 8 hours after birth. Feelings of faintness or BPM. During the 4th stage of labor, the maternal blood
dizziness are signs that should caution the nurse to be aware pressure, pulse, and respiration should be checked every 15
of the client’s safety. The nurse should advise the mother to minutes during the first hour. A rising pulse is an early sign of
get help the first few times the mother gets out of bed. excessive blood loss because the heart pumps faster to
Obtaining an H/H requires a physicians order. compensate for reduced blood volume. The blood pressure
4. Answer: 3. Ask the mother to urinate and empty her will fall as the blood volume diminishes, but a decreased
bladder. Before starting the fundal assessment, the nurse blood pressure would not be the earliest sign of hemorrhage.
should ask the mother to empty her bladder so that an A slight rise in temperature is normal. The respiratory rate is
accurate assessment can be done. When the nurse is increased slightly.
14. Answer: 1. Massage the fundus until it is firm. If the stimulation to the mother’s nipples affects hormonal levels
uterus is not contracted firmly, the first intervention is to and milk production.
massage the fundus until it is firm and to express clots that 25. Answer: 3. Teaching how to express her breasts in
may have accumulated in the uterus. Pushing on an a warm shower. Teaching the client how to express her
uncontracted uterus can invert the uterus and cause massive breasts in a warm shower aids with let-down and will give
hemorrhage. Elevating the client’s legs and encouraging the temporary relief. Ice can promote comfort by
client to void will not assist in managing uterine atony. If the vasoconstriction, numbing, and discouraging further letdown
uterus does not remain contracted as a result of the uterine of milk.
massage, the problem may be distended bladder and the 26. Answer: 1. Ask the client to empty her bladder. A
nurse should assist the mother to urinate, but this would not full bladder may displace the uterine fundus to the left or
be the initial action. right side of the abdomen. Catheterization is unnecessary
15. Answer: 2. Enlarged, hardened veins. Thrombosis of invasive if the woman can void on her own.
the superficial veins is usually accompanied by signs and 27. Answer: 3. Lower than before she became
symptoms of inflammation. These include swelling of the pregnant. PP insulin requirements are usually significantly
involved extremity and redness, tenderness, and warmth. lower than pre pregnancy requirements. Occasionally, clients
16. Answer: 4. “I need to stop breastfeeding until this may require little to no insulin during the first 24 to 48 hours
condition resolves.” In most cases, the mother can postpartum.
continue to breastfeed with both breasts. If the affected 28. Answer: 1. Fundus 1 cm above the umbilicus 1 hour
breast is too sore, the mother can pump the breast gently. postpartum. Within the first 12 hours postpartum, the
Regular emptying of the breast is important to prevent fundus usually is approximately 1 cm above the umbilicus.
abscess formation. Antibiotic therapy assists in resolving the The fundus should be below the umbilicus by PP day 3. The
mastitis within 24-48 hours. Additional supportive measures fundus shouldn’t be palpated in the abdomen after day 10.
include ice packs, breast supports, and analgesics. 29. Answer: 3. Multiple gestation. Multiple gestation,
17. Answer: 3. Hematuria, ecchymosis, and breastfeeding, multiparity, and conditions that cause
epistaxis. The treatment for DVT is anticoagulant therapy. overdistention of the uterus will increase the intensity of
The nurse assesses for bleeding, which is an adverse effect of after-pains. Bottle-feeding and diabetes aren’t directly
anticoagulants. This includes hematuria, ecchymosis, and associated with increasing severity of afterpains unless the
epistaxis. Dysuria and vertigo are not associated specifically client has delivered a macrosomic infant.
with bleeding. 30. Answer: 2. Days 3 to 10 PP. On the third and fourth PP
18. Answer: 1. Assess for hypovolemia and notify the days, the lochia becomes a pale pink or brown and contains
health care provider. Symptoms of hypovolemia include old blood, serum, leukocytes, and tissue debris. This type of
cool, clammy, pale skin, sensations of anxiety or impending lochia usually lasts until PP day 10. Lochia rubra usually last
doom, restlessness, and thirst. When these symptoms are for the first 3 to 4 days PP. Lochia alba, which contain
present, the nurse should further assess for hypovolemia and leukocytes, decidua, epithelial cells, mucus, and bacteria,
notify the health care provider. may continue for 2 to 6 weeks PP.
19. Answer: 3. Notify the physician. If the bleeding is 31. Answer: 1. Passive and dependant. During the taking
excessive, the cause may be laceration of the cervix or birth in phase, which usually lasts 1-3 days, the mother is passive
canal. Massaging the fundus if it is firm will not assist in and dependent and expresses her own needs rather than the
controlling the bleeding. Trendelenburg’s position is to be neonate’s needs. The taking hold phase usually lasts from
avoided because it may interfere with cardiac function. days 3-10 PP. During this stage, the mother strives for
20. 3. Activated partial thromboplastin independence and autonomy; she also becomes curious and
time. Anticoagulation therapy may be used to prevent the interested in the care of the baby and is most ready to learn.
extension of thrombus by delaying the clotting time of the 32. Answer: 3. Cervical laceration. Continuous seepage of
blood. Activated partial thromboplastin time should be blood may be due to cervical or vaginal lacerations if the
monitored, and a heparin dose should be adjusted to maintain uterus is firm and contracting. Retained placental fragments
a therapeutic level of 1.5 to 2.5 times the control. The and uterine atony may cause subinvolution of the uterus,
prothrombin time and the INR are used to monitor making it soft, boggy, and larger than expected. UTI won’t
coagulation time when warfarin (Coumadin) is used. cause vaginal bleeding, although hematuria may be present.
21. Answer: 2, 4, and 5. Mastitis are an infection of the 33. Answer: 4. Transitional milk. Transitional milk comes
lactating breast. Client instructions include resting during the after colostrum and usually lasts until 2 weeks PP.
acute phase, maintaining a fluid intake of at least 3 L a day, 34. Answer: 4. Uterine subinvolution. Late postpartum
and taking analgesics to relieve discomfort. Antibiotics may bleeding is often the result of subinvolution of the uterus.
be prescribed and are taken until the complete prescribed Retained products of conception or infection often cause
course is finished. They are not stopped when the soreness subinvolution. Cervical or perineal lacerations can cause an
subsides. Additional supportive measures include the use of immediate postpartum hemorrhage. A client with a clotting
moist heat or ice packs and wearing a supportive bra. deficiency may also have an immediate PP hemorrhage if the
Continued decompression of the breast by breastfeeding or deficiency isn’t corrected at the time of delivery.
pumping is important to empty the breast and prevent 35. Answer: 4. The client should avoid getting pregnant
formation of an abscess. for 3 months after the vaccine because the vaccine has
22. Answer: 2. Blood pressure. Methergine and pitocin are teratogenic effects. The client must understand that she
agents that are used to prevent or control postpartum must not become pregnant for 3 months after the vaccination
hemorrhage by contracting the uterus. They cause continuous because of its potential teratogenic effects. The rubella
uterine contractions and may elevate blood pressure. A vaccine is made from duck eggs so an allergic reaction may
priority nursing intervention is to check blood pressure. The occur in clients with egg allergies. The virus is not transmitted
physician should be notified if hypertension is present. into the breast milk, so clients may continue to breastfeed
23. Answer: 1. Peripheral vascular disease. These after the vaccination. Transient arthralgia and rash are
medications are avoided in clients with significant common adverse effects of the vaccine.
cardiovascular disease, peripheral disease, hypertension, 36. Answer: 2. Decrease. The placenta produces the
eclampsia, or preeclampsia. These conditions are worsened hormone human placental lactogen, an insulin antagonist.
by the vasoconstriction effects of these medications. After birth, the placenta, the major source of insulin
24. Answer: 1. Supplemental feedings with resistance, is gone. Insulin needs decrease and women with
formula. Routine formula supplementation may interfere with type 1 diabetes may only need one-half to two-thirds of the
establishing an adequate milk volume because decreased prenatal insulin during the first few PP days.
37. Answer: 4. Mothers with diabetes may breastfeed; alters the position of the fundus and makes the findings
insulin requirements may decrease from inaccurate. Although hands are washed before starting the
breastfeeding. Breastfeeding has an antidiabetogenic effect. check, clean (not sterile) gloves are put on just before the
Insulin needs are decreased because carbohydrates are used perineum and pad are assessed to protect from contact with
in milk production. Breastfeeding mothers are at a higher risk blood and secretions.
of hypoglycemia in the first PP days after birth because the 49. Answer: 4. Uses the peribottle to rinse upward into
glucose levels are lower. Mothers with diabetes should be her vagina. Responses 1, 2, and 3 are all appropriate
encouraged to breastfeed. measures. The peribottle should be used in a backward
38. Answer: 4. Taking-in phase. The taking-in phase direction over the perineum. The flow should never be
occurs in the first 24 hours after birth. The mother is directed upward into the vagina since debris would be forced
concerned with her own needs and requires support from staff upward into the uterus through the still-open cervix.
and relatives. The taking-hold phase occurs when the mother 50. Answer: 3. Massage the fundus every hour for the
is ready to take responsibility for her care as well as the first 24 hours following birth. The fundus should be
infants care. The letting-go phase begins several weeks later, massaged only when boggy or soft. Massaging a firm fundus
when the mother incorporates the new infant into the family could cause it to relax. Responses 1, 2, and 4 are all effective
unit. measures to enhance and maintain contraction of the uterus
39. Answer: 2. Rapid diuresis. In the early PP period, and to facilitate healing.
there’s an increase in the glomerular filtration rate and a drop 51. Answer: 3. Express a strong need to review events
in the progesterone levels, which result in rapid diuresis. and her behavior during the process of labor and
There should be no urinary urgency, though a woman may birth. One week after birth the woman should exhibit
feel anxious about voiding. There’s a minimal change in blood behaviors characteristic of the taking-hold stage as described
pressure following childbirth, and a residual decrease in GI in response 3. This stage lasts for as long as 4 to 5
motility. weeks after birth. Responses 1 and 2 are characteristic of the
40. Answer: 1. The client appears interested in learning taking-in stage, which lasts for the first few days after
about neonatal care. The third to tenth days of PP care are birth. Response 4 reflects the letting-go stage, which
the “taking-hold” phase, in which the new mother strives for indicates that psychosocial recovery is complete.
independence and is eager for her neonate. The other options 52. Answer: 4. Recognize this as a behavior of the
describe the phase in which the mother relives her birth taking-hold stage. Response 1 does not take into
experience. consideration the need for the new mother to be nurtured and
41. Answer: 3. Urine retention. Urine retention causes a have her needs met during the taking-in stage. The behavior
distended bladder to displace the uterus above the umbilicus described is typical of this stage and not a reflection of
and to the side, which prevents the uterus from contracting. ineffective attachment unless the behavior persists. Mothers
The uterus needs to remain contracted if bleeding is to stay need to reestablish their own well-being in order to effectively
within normal limits. Cervical and vaginal tears can cause PP care for their baby.
hemorrhage but are less common occurrences in the PP 53. Answer: 1. Having the children choose or make a
period. gift to give to the new baby upon its arrival
42. Answer: 4. Lochia rubra home. Special time should be set aside just for the other
43. Answer: 4. Multigravidas are at increased risk for children without interruption from the newborn. Someone
uterine atony. Multiple full-term pregnancies and deliveries other than the mother should carry the baby into the home so
result in overstretched uterine muscles that do not contract she can give full attention to greeting her other
efficiently and bleeding may ensue. children. Children should be actively involved in the care of
44. Answer: 1. Soft, non-tender; colostrum is the baby according to their ability without overwhelming
present. Breasts are essentially unchanged for the first two them.
to three days after birth. Colostrum is present and may leak 54. Answer: 2. Provide time for the mother to reflect on
from the nipples. the events of and her behavior during childbirth. The
45. Answer: 3. The expected weight loss immediately focus of the taking-in stage is nurturing the new mother by
after birth averages about 11 to 13 pounds. Prepregnant meeting her dependency needs for rest, comfort, hygiene,
weight is usually achieved by 2 to 3 months after birth, not and nutrition. Once they are met, she is more able to take an
within the 6-week postpartum period. Weight loss from active role, not only in her own care but also the care of her
diuresis, diaphoresis, and bleeding is about 9 pounds. Weight newborn. Women express a need to review their childbirth
loss continues during breastfeeding since fat stores developed experience and evaluate their performance. Short teaching
during pregnancy and extra calories consumed are used as sessions, using written materials to reinforce the content
part of the lactation process. presented, are a more effective approach.
46. Answer: 4. Pain in left calf with dorsiflexion of left 55. Answer: 3. Placement in a warm environment
foot. Responses 1 and 3 are expected related to circulatory
changes after birth. A temperature of 100.4°F in the first 24 NCLEX Exam: Newborn Nursing Care (50 Items)
hours is most likely indicative of dehydration which is easily
corrected by increasing oral fluid intake. The findings in 1. A nurse in a delivery room is assisting with the
response 4 indicate a positive Homan sign and are suggestive delivery of a newborn infant. After the delivery, the
of thrombophlebitis and should be investigated further. nurse prepares to prevent heat loss in the newborn
47. Answer: 2. Massage her fundus. A boggy or soft resulting from evaporation by:
fundus indicates that uterine atony is present. This is 1. Warming the crib pad
confirmed by the profuse lochia and passage of clots. The first 2. Turning on the overhead radiant warmer
action would be to massage the fundus until firm, followed by 3. Closing the doors to the room
3 and 4, especially if the fundus does not become or remain 4. Drying the infant in a warm blanket
firm with massage. There is no indication of a distended 2. A nurse is assessing a newborn infant following
bladder since the fundus is midline and below the umbilicus. circumcision and notes that the circumcised area is red
48. Answer: 1. Assist the woman into a lateral position with a small amount of bloody drainage. Which of the
with upper leg flexed forward to facilitate the following nursing actions would be most appropriate?
examination of her perineum. While the supine position is 1. Document the findings
best for examining the abdomen, the woman should keep her 2. Contact the physician
arms at her sides and slightly flex her knees in order to relax 3. Circle the amount of bloody drainage on the dressing and
abdominal muscles and facilitate palpation of the fundus. The reassess in 30 minutes
bladder should be emptied before the check. A full bladder 4. Reinforce the dressing
3. A nurse in the newborn nursery is monitoring a 1. Deltoid
preterm newborn infant for respiratory distress 2. Triceps
syndrome. Which assessment signs if noted in the 3. Vastus lateralis
newborn infant would alert the nurse to the possibility 4. Biceps
of this syndrome? 11. A nursing instructor asks a nursing student to
1. Hypotension and Bradycardia describe the procedure for administering erythromycin
2. Tachypnea and retractions ointment into the eyes if a neonate. The instructor
3. Acrocyanosis and grunting determines that the student needs to research this
4. The presence of a barrel chest with grunting procedure further if the student states:
4. A nurse in a newborn nursery is performing an 1. “I will cleanse the neonate’s eyes before instilling
assessment of a newborn infant. The nurse is preparing ointment.”
to measure the head circumference of the infant. The 2. “I will flush the eyes after instilling the ointment.”
nurse would most appropriately: 3. “I will instill the eye ointment into each of the neonate’s
1. Wrap the tape measure around the infant’s head and conjunctival sacs within one hour after birth.”
measure just above the eyebrows. 4. “Administration of the eye ointment may be delayed until
2. Place the tape measure under the infants head at the base an hour or so after birth so that eye contact and parent-infant
of the skull and wrap around to the front just above the eyes attachment and bonding can occur.”
3. Place the tape measure under the infants head, wrap 12. A baby is born precipitously in the ER. The nurses
around the occiput, and measure just above the eyes initial action should be to:
4. Place the tape measure at the back of the infant’s head, 1. Establish an airway for the baby
wrap around across the ears, and measure across the infant’s 2. Ascertain the condition of the fundus
mouth. 3. Quickly tie and cut the umbilical cord
5. A postpartum nurse is providing instructions to the 4. Move mother and baby to the birthing unit
mother of a newborn infant with hyperbilirubinemia 13. The primary critical observation for Apgar scoring is
who is being breastfed. The nurse provides which most the:
appropriate instructions to the mother? 1. Heart rate
1. Switch to bottle feeding the baby for 2 weeks 2. Respiratory rate
2. Stop the breast feedings and switch to bottle-feeding 3. Presence of meconium
permanently 4. Evaluation of the Moro reflex
3. Feed the newborn infant less frequently 14. When performing a newborn assessment, the nurse
4. Continue to breast-feed every 2-4 hours should measure the vital signs in the following
6. A nurse on the newborn nursery floor is caring for a sequence:
neonate. On assessment the infant is exhibiting signs 1. Pulse, respirations, temperature
of cyanosis, tachypnea, nasal flaring, and grunting. 2. Temperature, pulse, respirations
Respiratory distress syndrome is diagnosed, and the 3. Respirations, temperature, pulse
physician prescribes surfactant replacement therapy. 4. Respirations, pulse, temperature
The nurse would prepare to administer this therapy by: 15. Within 3 minutes after birth the normal heart rate
1. Subcutaneous injection of the infant may range between:
2. Intravenous injection 1. 100 and 180
3. Instillation of the preparation into the lungs through an 2. 130 and 170
endotracheal tube 3. 120 and 160
4. Intramuscular injection 4. 100 and 130
7. A nurse is assessing a newborn infant who was born 16. The expected respiratory rate of a neonate within 3
to a mother who is addicted to drugs. Which of the minutes of birth may be as high as:
following assessment findings would the nurse expect 1. 50
to note during the assessment of this newborn? 2. 60
1. Sleepiness 3. 80
2. Cuddles when being held 4. 100
3. Lethargy 17. The nurse is aware that a healthy newborn’s
4. Incessant crying respirations are:
8. A nurse prepares to administer a vitamin K injection 1. Regular, abdominal, 40-50 per minute, deep
to a newborn infant. The mother asks the nurse why 2. Irregular, abdominal, 30-60 per minute, shallow
her newborn infant needs the injection. The best 3. Irregular, initiated by chest wall, 30-60 per minute, deep
response by the nurse would be: 4. Regular, initiated by the chest wall, 40-60 per minute,
1. “You infant needs vitamin K to develop immunity.” shallow
2. “The vitamin K will protect your infant from being 18. To help limit the development of hyperbilirubinemia
jaundiced.” in the neonate, the plan of care should include:
3. “Newborn infants are deficient in vitamin K, and this 1. Monitoring for the passage of meconium each shift
injection prevents your infant from abnormal bleeding.” 2. Instituting phototherapy for 30 minutes every 6 hours
4. “Newborn infants have sterile bowels, and vitamin K 3. Substituting breastfeeding for formula during the 2nd day
promotes the growth of bacteria in the bowel.” after birth
9. A nurse in a newborn nursery receives a phone call 4. Supplementing breastfeeding with glucose water during the
to prepare for the admission of a 43-week-gestation first 24 hours
newborn with Apgar scores of 1 and 4. In planning for 19. A newborn has small, whitish, pinpoint spots over
the admission of this infant, the nurse’s highest priority the nose, which the nurse knows are caused by
should be to: retained sebaceous secretions. When charting this
1. Connect the resuscitation bag to the oxygen outlet observation, the nurse identifies it as:
2. Turn on the apnea and cardiorespiratory monitors 1. Milia
3. Set up the intravenous line with 5% dextrose in water 2. Lanugo
4. Set the radiant warmer control temperature at 36.5* C 3. Whiteheads
(97.6*F) 4. Mongolian spots
10. Vitamin K is prescribed for a neonate. A nurse 20. When newborns have been on formula for 36-48
prepares to administer the medication in which muscle hours, they should have a:
site?
1. Screening for PKU 30. A neonate has been diagnosed with caput
2. Vitamin K injection succedaneum. Which statement is correct about this
3. Test for necrotizing enterocolitis condition?
4. Heel stick for blood glucose level 1. It usually resolves in 3-6 weeks
21. The nurse decides on a teaching plan for a new 2. It doesn’t cross the cranial suture line
mother and her infant. The plan should include: 3. It’s a collection of blood between the skull and the
1. Discussing the matter with her in a non-threatening periosteum
manner 4. It involves swelling of tissue over the presenting part of the
2. Showing by example and explanation how to care for the presenting head
infant 31. The most common neonatal sepsis and meningitis
3. Setting up a schedule for teaching the mother how to care infections seen within 24 hours after birth are caused
for her baby by which organism?
4. Supplying the emotional support to the mother and 1. Candida albicans
encouraging her independence 2. Chlamydia trachomatis
22. Which action best explains the main role of 3. Escherichia coli
surfactant in the neonate? 4. Group B beta-hemolytic streptococci
1. Assists with ciliary body maturation in the upper airways 32. When attempting to interact with a neonate
2. Helps maintain a rhythmic breathing pattern experiencing drug withdrawal, which behavior would
3. Promotes clearing mucus from the respiratory tract indicate that the neonate is willing to interact?
4. Helps the lungs remain expanded after the initiation of 1. Gaze aversion
breathing 2. Hiccups
23. While assessing a 2-hour old neonate, the nurse 3. Quiet alert state
observes the neonate to have acrocyanosis. Which of 4. Yawning
the following nursing actions should be performed 33. When teaching umbilical cord care to a new
initially? mother, the nurse would include which information?
1. Activate the code blue or emergency system 1. Apply peroxide to the cord with each diaper change
2. Do nothing because acrocyanosis is normal in the neonate 2. Cover the cord with petroleum jelly after bathing
3. Immediately take the newborn’s temperature according to 3. Keep the cord dry and open to air
hospital policy 4. Wash the cord with soap and water each day during a tub
4. Notify the physician of the need for a cardiac consult bath
24. The nurse is aware that a neonate of a mother with 34. A mother of a term neonate asks what the thick,
diabetes is at risk for what complication? white, cheesy coating is on his skin. Which correctly
1. Anemia describes this finding?
2. Hypoglycemia 1. Lanugo
3. Nitrogen loss 2. Milia
4. Thrombosis 3. Nevus flammeus
25. A client with group AB blood whose husband has 4. Vernix
group O has just given birth. The major sign of ABO 35. Which condition or treatment best ensures lung
blood incompatibility in the neonate is which maturity in an infant?
complication or test result? 1. Meconium in the amniotic fluid
1. Negative Coombs test 2. Glucocorticoid treatment just before delivery
2. Bleeding from the nose and ear 3. Lecithin to sphingomyelin ratio more than 2:1
3. Jaundice after the first 24 hours of life 4. Absence of phosphatidylglycerol in amniotic fluid
4. Jaundice within the first 24 hours of life 36. When performing nursing care for a neonate after a
26. A client has just given birth at 42 weeks’ gestation. birth, which intervention has the highest nursing
When assessing the neonate, which physical finding is priority?
expected? 1. Obtain a dextrostix
1. A sleepy, lethargic baby 2. Give the initial bath
2. Lanugo covering the body 3. Give the vitamin K injection
3. Desquamation of the epidermis 4. Cover the neonates head with a cap
4. Vernix caseosa covering the body 37. When performing an assessment on a neonate,
27. After reviewing the client’s maternal history of which assessment finding is most suggestive of
magnesium sulfate during labor, which condition would hypothermia?
the nurse anticipate as a potential problem in the 1. Bradycardia
neonate? 2. Hyperglycemia
1. Hypoglycemia 3. Metabolic alkalosis
2. Jitteriness 4. Shivering
3. Respiratory depression 38. A woman delivers a 3.250 g neonate at 42 weeks’
4. Tachycardia gestation. Which physical finding is expected during an
28. Neonates of mothers with diabetes are at risk for examination if this neonate?
which complication following birth? 1. Abundant lanugo
1. Atelectasis 2. Absence of sole creases
2. Microcephaly 3. Breast bud of 1-2 mm in diameter
3. Pneumothorax 4. Leathery, cracked, and wrinkled skin
4. Macrosomia 39. A healthy term neonate born by C-section was
29. By keeping the nursery temperature warm and admitted to the transitional nursery 30 minutes ago
wrapping the neonate in blankets, the nurse is and placed under a radiant warmer. The neonate has
preventing which type of heat loss? an axillary temperature of 99.5oF, a respiratory rate of
1. Conduction 80 breaths/minute, and a heel stick glucose value of 60
2. Convection mg/dl. Which action should the nurse take?
3. Evaporation 1. Wrap the neonate warmly and place her in an open crib
4. Radiation 2. Administer an oral glucose feeding of 10% dextrose in
water
3. Increase the temperature setting on the radiant warmer carbohydrates like cereal.”
4. Obtain an order for IV fluid administration 4. “If you want him to gain weight, just double his daily
40. Which neonatal behavior is most commonly intake of formula.”
associated with fetal alcohol syndrome (FAS)? 48. The nurse instructs a primipara about safety
1. Hypoactivity considerations for the neonate. The nurse determines
2. High birth weight that the client does not understand the instructions
3. Poor wake and sleep patterns when she says
4. High threshold of stimulation 1. “All neonates should be in an approved car seat when in an
41. Which of the following behaviors would indicate automobile.”
that a client was bonding with her baby? 2. “It’s acceptable to prop the infant’s bottle once in a while.”
1. The client asks her husband to give the baby a bottle of 3. “Pillows should not be used in the infant’s crib.”
water. 4. “Infants should never be left unattended on an unguarded
2. The client talks to the baby and picks him up when he surface.”
cries. 49. The nurse manager is presenting education to her
3. The client feeds the baby every three hours. staff to promote consistency in the interventions used
4. The client asks the nurse to recommend a good child care with lactating mothers. She emphasizes that the
manual. optimum time to initiate lactation is
42. A newborn’s mother is alarmed to find small 1. as soon as possible after the infant’s birth.
amounts of blood on her infant girl’s diaper. When the 2. after the mother has rested for 4-6 hours.
nurse checks the infant’s urine it is straw colored and 3. during the infant’s second period of reactivity.
has no offensive odor. Which explanation to the 4. after the infant has taken sterile water without
newborn’s mother is most appropriate? complications.
1. “It appears your baby has a kidney infection” 50. The nurse is preparing to discharge a multipara 24
2. “Breast-fed babies often experience this type of bleeding hours after a vaginal delivery. The client is breast-
problem due to lack of vitamin C in the breast milk” feeding her newborn. The nurse instructs the client
3. “The baby probably passed a small kidney stone” that if engorgement occurs the client should
4. “Some infants experience menstruation like bleeding when 1. wear a tight fitting bra or breast binder.
hormones from the mother are not available” 2. apply warm, moist heat to the breasts.
43. An insulin-dependent diabetic delivered a 10-pound 3. contact the nurse midwife for a lactation suppressant.
male. When the baby is brought to the nursery, the 4. restrict fluid intake to 1000 ml. daily .
priority of care is to
1. clean the umbilical cord with Betadine to prevent infection Answers and Rationale
2. give the baby a bath
3. call the laboratory to collect a PKU screening test 1. Answer: 4. Drying the infant in a warm
4. check the baby’s serum glucose level and administer blanket. Evaporation of moisture from a wet body dissipates
glucose if < 40 mg/dL heat along with the moisture. Keeping the newborn dry by
44. Soon after delivery a neonate is admitted to the drying the wet newborn infant will prevent hypothermia via
central nursery. The nursery nurse begins the initial evaporation.
assessment by 2. Answer: 1. Document the findings. The penis is
1. auscultate bowel sounds. normally red during the healing process. A yellow
2. determining chest circumference. exudate may be noted in 24 hours, and this is a part of
3. inspecting the posture, color, and respiratory effort. normal healing. The nurse would expect that the area would
4. checking for identifying birthmarks. be red with a small amount of bloody drainage. If the
45. The home health nurse visits the Cox family 2 bleeding is excessive, the nurse would apply gentle pressure
weeks after hospital discharge. She observes that the with sterile gauze. If bleeding is not controlled, then the blood
umbilical cord has dried and fallen off. The area vessel may need to be ligated, and the nurse would contact
appears healed with no drainage or erythema present. the physician. Because the findings identified in the question
The mother can be instructed to are normal, the nurse would document the assessment.
1. cover the umbilicus with a band-aid. 3. Answer: 2. Acrocyanosis and grunting. The infant with
2. continue to clean the stump with alcohol for one week. respiratory distress syndrome may present with signs of
3. apply an antibiotic ointment to the stump. cyanosis, tachypnea or apnea, nasal flaring, chest wall
4. give him a bath in an infant tub now. retractions, or audible grunts.
46. A neonate is admitted to a hospital’s central 4. Answer: 3. Place the tape measure under the infants
nursery. The neonate’s vital signs are: temperature = head, wrap around the occiput, and measure just above
96.5 degrees F., heart rate = 120 bpm, and respirations the eyes. To measure the head circumference, the nurse
= 40/minute. The infant is pink with slight should place the tape measure under the infant’s head, wrap
acrocyanosis. The priority nursing diagnosis for the the tape around the occiput, and measure just above the
neonate is eyebrows so that the largest area of the occiput is included.
1. Ineffective thermoregulation related to 5. Answer: 4. Continue to breastfeed every 2-4
fluctuating environmental temperatures. hours. Breast feeding should be initiated within 2 hours after
2. Potential for infection related to lack of immunity. birth and every 2-4 hours thereafter. The other options are
3. Altered nutrition, less than body requirements related to not necessary.
diminished sucking reflex. 6. Answer: 3. Instillation of the preparation into the
4. Altered elimination pattern related to lack of nourishment. lungs through an endotracheal tube. The aim of therapy
47. The nurse hears the mother of a 5-pound neonate in RDS is to support the disease until the disease runs its
telling a friend on the telephone, “As soon as I get course with the subsequent development of surfactant. The
home, I’ll give him some cereal to get him to gain infant may benefit from surfactant replacement therapy. In
weight?” The nurse recognizes the need for further surfactant replacement, an exogenous surfactant preparation
instruction about infant feeding and tells her is instilled into the lungs through an endotracheal tube.
1. “If you give the baby cereal, be sure to use Rice to prevent 7. Answer: 4. Incessant crying. A newborn infant born to a
allergy.” woman using drugs is irritable. The infant is overloaded easily
2. “The baby is not able to swallow cereal, because he is too by sensory stimulation. The infant may cry incessantly and
small.” posture rather than cuddle when being held.
3. “The infant’s digestive tract cannot handle complex
8. Answer: 3. “Newborn infants are deficient in vitamin transferred to the fetus across the placenta. The neonate’s
K, and this injection prevents your infant from liver cannot initially adjust to the changing glucose levels
abnormal bleeding.” Vitamin K is necessary for the body to after birth. This may result in an overabundance of insulin in
synthesize coagulation factors. Vitamin K is administered to the neonate, resulting in hypoglycemia.
the newborn infant to prevent abnormal bleeding. Newborn 25. Answer: 4. Jaundice within the first 24 hours of
infants are vitamin K deficient because the bowel does not life. The neonate with ABO blood incompatibility with its
have the bacteria necessary for synthesizing fat-soluble mother will have jaundice (pathologic) within the first 24
vitamin K. The infant’s bowel does not have support the hours of life. The neonate would have a positive Coombs test
production of vitamin K until bacteria adequately colonizes it result.
by food ingestion. 26. Answer: 3. Desquamation of the epidermis. Postdate
9. Answer: 1. Connect the resuscitation bag to the fetuses lose the vernix caseosa, and the epidermis may
oxygen outlet. The highest priority on admission to the become desquamated. These neonates are usually very alert.
nursery for a newborn with low Apgar scores is airway, which Lanugo is missing in the postdate neonate.
would involve preparing respiratory resuscitation equipment. 27. Answer: 3. Respiratory depression. Magnesium
The other options are also important, although they are of sulfate crosses the placenta and adverse neonatal effects are
lower priority. respiratory depression, hypotonia, and Bradycardia.
10. Answer: 3. Vastus lateralis. 28. Answer: 4. Macrosomia. Neonates of mothers with
11. Answer: 2. “I will flush the eyes after instilling the diabetes are at increased risk for macrosomia (excessive fetal
ointment.” Eye prophylaxis protects the neonate growth) as a result of the combination of the increased supply
against Neisseria gonorrhoeae and Chlamydia trachomatis. of maternal glucose and an increase in fetal insulin.
The eyes are not flushed after instillation of the medication 29. Answer: 2. Convection. Convection heat loss is the flow
because the flush will wash away the administered of heat from the body surface to the cooler air.
medication. 30. Answer: 4. It involves swelling of tissue over the
12. Answer: 1. Establish an airway for the baby. The presenting part of the presenting head. Caput
nurse should position the baby with head lower than chest succedaneum is the swelling of tissue over the presenting
and rub the infant’s back to stimulate crying to promote part of the fetal scalp due to sustained pressure; it resolves in
oxygenation. There is no haste in cutting the cord. 3-4 days.
13. Answer: 1. Heart rate. The heart rate is vital for life 31. Answer: 4. Group B beta-hemolytic
and is the most critical observation in Apgar scoring. streptococci. Transmission of Group B beta-hemolytic
Respiratory effect rather than rate is included in the Apgar streptococci to the fetus results in respiratory distress that
score; the rate is very erratic. can rapidly lead to septic shock.
14. Answer: 4. Respirations, pulse, temperature. This 32. Answer: 3. Quiet alert state. When caring for a
sequence is least disturbing. Touching with the stethoscope neonate experiencing drug withdrawal, the nurse needs to be
and inserting the thermometer increase anxiety and elevate alert for distress signals from the neonate. Stimuli should be
vital signs. introduced one at a time when the neonate is in a quiet and
15. Answer: 3. 120 and 160. The heart rate varies with alert state. Gaze aversion, yawning, sneezing, hiccups, and
activity; crying will increase the rate, whereas deep sleep will body arching are distress signals that the neonate cannot
lower it; a rate between 120 and 160 is expected. handle stimuli at that time.
16. Answer: 2. 60. The respiratory rate is associated with 33. Answer: 3. Keep the cord dry and open to
activity and can be as rapid as 60 breaths per minute; over air. Keeping the cord dry and open to air helps reduce
60 breaths per minute are considered tachypneic in the infection and hastens drying.
infant. 34. Answer: 4. Vernix.
17. Answer: 2. Irregular, abdominal, 30-60 per minute, 35. Answer: 3. Lecithin to sphingomyelin ratio more
shallow. Normally the newborn’s breathing is abdominal and than 2:1. Lecithin and sphingomyelin are phospholipids that
irregular in depth and rhythm; the rate ranges from 30-60 help compose surfactant in the lungs; lecithin peaks at 36
breaths per minute. weeks and sphingomyelin concentrations remain stable.
18. Answer: 1. Monitoring for the passage of meconium 36. Answer: 4. Cover the neonates head with a
each shift. Bilirubin is excreted via the GI tract; if meconium cap. Covering the neonates head with a cap helps prevent
is retained, the bilirubin is reabsorbed. cold stress due to excessive evaporative heat loss from the
19. Answer: 1. Milia. Milia occur commonly, are not neonate’s wet head. Vitamin K can be given up to 4 hours
indicative of any illness, and eventually disappear. after birth.
20. Answer: 1. Screening for PKU. By now the newborn 37. Answer: 1. Bradycardia. Hypothermic neonates
will have ingested an ample amount of the amino acid become bradycardic proportional to the degree of core
phenylalanine, which, if not metabolized because of a lack of temperature. Hypoglycemia is seen in hypothermic neonates.
the liver enzyme, can deposit injurious metabolites into the 38. Answer: 4. Leathery, cracked, and wrinkled
bloodstream and brain; early detection can determine if the skin. Neonatal skin thickens with maturity and is often
liver enzyme is absent. peeling by post term.
21. Answer: 2. Showing by example and explanation 39. Answer: 4. Obtain an order for IV fluid
how to care for the infant. Teaching the mother by administration. Assessment findings indicate that the
example is a non-threatening approach that allows her to neonate is in respiratory distress—most likely from transient
proceed at her own pace. tachypnea, which is common after cesarean delivery. A
22. Answer: 4. Helps the lungs remain expanded after neonate with a rate of 80 breaths a minute shouldn’t be fed
the initiation of breathing. Surfactant works by reducing but should receive IV fluids until the respiratory rate returns
surface tension in the lung. Surfactant allows the lung to to normal. To allow for close observation for worsening
remain slightly expanded, decreasing the amount of work respiratory distress, the neonate should be kept unclothed in
required for inspiration. the radiant warmer.
23. Answer: 2. Do nothing because acrocyanosis is 40. Answer: 3. Poor wake and sleep patterns. Altered
normal in the neonate. Acrocyanosis, or bluish discoloration sleep patterns are caused by disturbances in the CNS from
of the hands and feet in the neonate (also called peripheral alcohol exposure in utero. Hyperactivity is a characteristic
cyanosis), is a normal finding and shouldn’t last more than 24 generally noted. Low birth weight is a physical defect seen in
hours after birth. neonates with FAS. Neonates with FAS generally have a low
24. Answer: 2. Hypoglycemia. Neonates of mothers with threshold for stimulation.
diabetes are at risk for hypoglycemia due to increased insulin 41. Answer: 2. The client talks to the baby and picks
levels. During gestation, an increased amount of glucose is him up when he cries.
42. Answer: 4. “Some infants experience menstruation needs. During adolescence, psychosocial development
like bleeding when hormones from the mother are not focuses on:
available”. a. Becoming industrious
43. Answer: 4. check the baby’s serum glucose level b. Establishing an identity
and administer glucose if < 40 mg/dL. c. Achieving intimacy
44. Answer: 3. inspecting the posture, color, and d. Developing initiative
respiratory effort. 9. When developing a plan care for a hospitalized child,
45. Answer:4. give him a bath in an infant tub now. nurse Mica knows that children in which age group are
46. Answer: 1. Ineffective thermoregulation related to most likely to view illness as a punishment for
fluctuating environmental temperatures. misdeeds?
47. Answer: 3. “The infant’s digestive tract cannot a. Infancy
handle complex carbohydrates like cereal.” b. Preschool age
48. Answer: 2. “It’s acceptable to prop the infant’s c. School age
bottle once in a while.” d. Adolescence
49. Answer: 1. as soon as possible after the infant’s 10. Nurse Sunshine suspects that a child, age 4, is
birth. being neglected physically. To best assess the child’s
50. Answer: 2. apply warm, moist heat to the breasts. nutritional status, the nurse should ask the parents
which question?
NCLEX Exam: Pediatric Nursing 1 (50 Items) a. “Has your child always been so thin?”
b. “Is your child a picky eater?”
1. Molly, with suspected rheumatic fever, is admitted to c. “What did your child eat for breakfast?”
the pediatric unit. When obtaining the child’s history, d. “Do you think your child eats enough?”
the nurse considers which information to be most 11. A female child, age 2, is brought to the emergency
important? department after ingesting an unknown number of
a. A fever that started 3 days ago aspirin tablets about 30 minutes earlier. On entering
b. Lack of interest in food the examination room, the child is crying and clinging
c. A recent episode of pharyngitis to the mother. Which data should the nurse obtain
d. Vomiting for 2 days first?
2. Nurse Analiza is administering a medication via the a. Heart rate, respiratory rate, and blood pressure
intraosseous route to a child. Intraosseous drug b. Recent exposure to communicable diseases
administration is typically used when a child is: c. Number of immunizations received
a. Under age 3 d. Height and weight
b. Over age 3 12. A mother asks the nurse how to handle her 5-year-
c. Critically ill and under age 3 old child, who recently started wetting the pants after
d. Critically ill and over age 3 being completely toilet trained. The child just started
3. When assessing a child’s cultural background, the attending nursery school 2 days a week. Which
nurse in charge should keep in mind that: principle should guide the nurse’s response?
a. Cultural background usually has little bearing on a family’s a. The child forgets previously learned skills
health practices b. The child experiences growth while regressing, regrouping,
b. Physical characteristics mark the child as part of a and then progressing
particular culture c. The parents may refer less mature behaviors
c. Heritage dictates a group’s shared values d. The child returns to a level of behavior that increases the
d. Behavioral patterns are passed from one generation to the sense of security.
next 13. A female child, age 6, is brought to the health clinic
4. While examining a 2-year-old child, the nurse in for a routine checkup. To assess the child’s vision, the
charge sees that the anterior fontanel is open. The nurse should ask:
nurse should: a. “Do you have any problems seeing different colors?”
a. Notify the doctor b. “Do you have trouble seeing at night?”
b. Look for other signs of abuse c. “Do you have problems with glare?”
c. Recognize this as a normal finding d. “How are you doing in school?”
d. Ask about a family history of Tay-Sachs disease 14. During a well-baby visit, Liza asks the nurse when
5. The nurse is aware that the most common she should start giving her infant solid foods. The nurse
assessment finding in a child with ulcerative colitis is: should instruct her to introduce which solid food first?
a. Intense abdominal cramps a. Applesauce
b. Profuse diarrhea b. Egg whites
c. Anal fissures c. Rice cereal
d. Abdominal distention d. Yogurt
6. When administering an I.M. injection to an infant, 15. To decrease the likelihood of bradyarrhythmias in
the nurse in charge should use which site? children during endotracheal intubation,
a. Deltoid succinylcholine (Anectine) is used with which of the
b. Dorsogluteal following agents?
c. Ventrogluteal a. Epinephrine (Adrenalin)
d. Vastus lateralis b. Isoproterenol (Isuprel)
7. A child with a poor nutritional status and weight loss c. Atropine sulfate
is at risk for a negative nitrogen balance. To help d. Lidocaine hydrochloride (Xylocaine)
diagnose this problem, the nurse in charge anticipates 16. A 1-year-and 2-month-old child weighing 26 lb
that the doctor will order which laboratory test? (11.8 kg) is admitted for traction to treat congenital
a. Total iron-binding capacity hip dislocation. When preparing the patient’s room, the
b. Hemoglobin nurse anticipates using which traction system?
c. Total protein a. Bryant’s traction
d. Serum transferrin b. Buck’s extension traction
8. When developing a plan of care for a male c. Overhead suspension traction
adolescent, the nurse considers the child’s psychosocial d. 90-90 traction
17. Hannah, age 12, is 7 months pregnant. When c. Moderately retarded but trainable
teaching parenting skills to an adolescent, the nurse d. Completely dependent on others for care
knows that which teaching strategy is least effective? 27. Mandy, age 12, is brought to the clinic for
a. Providing a one-on-one demonstration and requesting a evaluation for a suspected eating disorder. To best
return demonstration, using a live infant model assess the effects of role and relationship patterns on
b. Initiating a teenage parent support group with first – and – the child’s nutritional intake, the nurse should ask:
second-time mothers a. “What activities do you engage in during the day?”
c. Using audiovisual aids that show discussions of feelings and b. “Do you have any allergies to foods?”
skills c. “Do you like yourself physically?”
d. Providing age-appropriate reading materials d. “What kinds of food do you like to eat?”
18. When performing a physical examination on an 28. Sudden infant death syndrome (SIDS) is one of the
infant, the nurse in charge notes abnormally low-set most common causes of death in infants. At what age is
ears. This findings is associated with: the diagnosis of SIDS most likely?
a. Otogenous tetanus a. At 1 to 2 years of age
b. Tracheoesophageal fistula b. At I week to 1 year of age, peaking at 2 to 4 months
c. Congenital heart defects c. At 6 months to 1 year of age, peaking at 10 months
d. Renal anomalies d. At 6 to 8 weeks of age
19. Nurse Walter should expect a 3-year-old child to be 29. When evaluating a severely depressed adolescent,
able to perform which action? the nurse knows that one indicator of a high risk for
a. Ride a tricycle suicide is:
b. Tie the shoelaces a. Depression
c. Roller-skates b. Excessive sleepiness
d. Jump rope c. A history of cocaine use
20. Nurse Kim is teaching a group of parents about d. A preoccupation with death
otitis media. When discussing why children are 30. A child is diagnosed with Wilms’ tumor. During
predisposed to this disorder, the nurse should mention assessment, the nurse in charge expects to detect:
the significance of which anatomical feature? a. Gross hematuria
a. Eustachian tubes b. Dysuria
b. Nasopharynx c. Nausea and vomiting
c. Tympanic membrane d. An abdominal mass
d. External ear canal 31. Which of the following would be inappropriate
21. The nurse is evaluating a female child with acute when administering chemotherapy to a child?
poststreptococcal glomerulonephritis for signs of a. Monitoring the child for both general and specific adverse
improvement. Which finding typically is the earliest effects
sign of improvement? b. Observing the child for 10 minutes to note for signs of
a. Increased urine output anaphylaxis
b. Increased appetite c. Administering medication through a free-flowing
c. Increased energy level intravenous line
d. Decreased diarrhea d. Assessing for signs of infusion infiltration and irritation
22. Dr. Jones prescribes corticosteroids for a child with 32. Which of the following is the best method for
nephritic syndrome. What is the primary purpose of performing a physical examination on a toddler
administering corticosteroids to this child? a. From head to toe
a. To increase blood pressure b. Distally to proximally
b. To reduce inflammation c. From abdomen to toes, the to head
c. To decrease proteinuria d. From least to most intrusive
d. To prevent infection 33. Which of the following organisms is responsible for
23. Parents bring their infant to the clinic, seeking the development of rheumatic fever?
treatment for vomiting and diarrhea that has lasted for a. Streptococcal pneumonia
2 days. On assessment, the nurse in charge detects dry b. Haemophilus influenza
mucous membranes and lethargy. What other findings c. Group A β-hemolytic streptococcus
suggests a fluid volume deficit? d. Staphylococcus aureus
a. A sunken fontanel 34. Which of the following is most likely associated
b. Decreased pulse rate with a cerebrovascular accident (CVA) resulting from
c. Increased blood pressure congenital heart disease?
d. Low urine specific gravity a. Polycythemia
24. How should the nurse prepare a suspension before b. Cardiomyopathy
administration? c. Endocarditis
a. By diluting it with normal saline solution d. Low blood pressure
b. By diluting it with 5% dextrose solution 35. How does the nurse appropriately administer
c. By shaking it so that all the drug particles are dispersed mycostatin suspension in an infant?
uniformly a. Have the infant drink water, and then administer
d. By crushing remaining particles with a mortar and pestle mycostatin in a syringe
25. What should be the initial bolus of crystalloid fluid b. Place mycostatin on the nipple of the feeding bottle and
replacement for a pediatric patient in shock? have the infant suck it
a. 20 ml/kg c. Mix mycostatin with formula
b. 10 ml/kg d. Swab mycostatin on the affected areas
c. 30 ml/kg 36. A mother tells the nurse that she is very worried
d. 15 ml/kg because her 2-year old child does not finish his meals.
26. Lily , age 5, with intelligence quotient of 65 is What should the nurse advise the mother?
admitted to the hospital for evaluation. When planning a. make the child seat with the family in the dining room until
care, the nurse should keep in mind that this child is: he finishes his meal
a. Within the lower range of normal intelligence b. provide quiet environment for the child before meals
b. Mildly retarded but educable c. do not give snacks to the child before meals
d. put the child on a chair and feed him
37. The nurse is assessing a newborn who had b. semi-fowler’s so she can watch TV for five hours and be
undergone vaginal delivery. Which of the following entertained.
findings is least likely to be observed in a normal c. supine for several hours, to prevent headache.
newborn? d. on her right sides to encourage return of CSF
a. uneven head shape 46. Buck’s traction with a 10 lb. weight is securing a
b. respirations are irregular, abdominal, 30-60 bpm patient’s leg while she is waiting for surgery to repair a
c. (+) moro reflex hip fracture. It is important to check circulation-
d. heart rate is 80 bpm sensation-movement:
38. Which of the following situations increase risk of a. every shift.
lead poisoning in children? b. every day.
a. playing in the park with heavy traffic and with many c. every 4 hours.
vehicles passing by d. every 15 minutes.
b. playing sand in the park 47. Kim is using bronchodilators for asthma. The side
c. playing plastic balls with other children effects of these drugs that you need to monitor this
d. playing with stuffed toys at home patient for include:
39. An inborn error of metabolism that causes a. tachycardia, nausea, vomiting, heart palpitations, inability
premature destruction of RBC? to sleep, restlessness, and seizures.
a. G6PD b. tachycardia, headache, dyspnea, temp . 101 F, and
b. Hemocystinuria wheezing.
c. Phenylketonuria c. blurred vision, tachycardia, hypertension, headache,
d. Celiac Disease insomnia, and oliguria.
40. Which of the following blood study results would d. restlessness, insomnia, blurred vision, hypertension, chest
the nurse expect as most likely when caring for the pain, and muscle weakness.
child with iron deficiency anemia? 48. The adolescent patient has symptoms of
a. Increased hemoglobin meningitis: nuchal rigidity, fever, vomiting, and
b. Normal hematocrit lethargy. The nurse knows to prepare for the following
c. Decreased mean corpuscular volume (MCV) test:
d. Normal total iron-binding capacity (TIBC) a. blood culture.
41. The nurse answers a call bell and finds a frightened b. throat and ear culture.
mother whose child, the patient, is having a seizure. c. CAT scan.
Which of these actions should the nurse take? d. lumbar puncture.
a. The nurse should insert a padded tongue blade in the 49. The nurse is drawing blood from the diabetic
patient’s mouth to prevent the child from swallowing or patient for a glycosylated hemoglobin test. She
choking on his tongue. explains to the woman that the test is used to
b. The nurse should help the mother restrain the child to determine:
prevent him from injuring himself. a. the highest glucose level in the past week.
c. The nurse should call the operator to page for seizure b. her insulin level.
assistance. c. glucose levels over the past several months.
d. The nurse should clear the area and position the client d. her usual fasting glucose level.
safely. 50. The twelve-year-old boy has fractured his arm
42. At the community center, the nurse leads an because of a fall from his bike. After the injury has
adolescent health information group, which often been casted, the nurse knows it is most important to
expands into other areas of discussion. She knows that perform all of the following assessments on the area
these youths are trying to find out “who they are,” and distal to the injury except:
discussion often focuses on which directions they want a. capillary refill.
to take in school and life, as well as peer relationships. b. radial and ulnar pulse.
According to Erikson, this stage is known as: c. finger movement
a. identity vs. role confusion. d. skin integrity
b. adolescent rebellion.
c. career experimentation. Answers and Rationale
d. relationship testing 1. Answer C.
43. The nurse is assessing a 9-month-old boy for a A recent episode of pharyngitis is the most important factor in
well-baby check up. Which of the following establishing the diagnosis of rheumatic fever. Although the
observations would be of most concern? child may have a history of fever or vomiting or lack interest
a. The baby cannot say “mama” when he wants his mother. in food, these findings are not specific to rheumatic fever.
b. The mother has not given him finger foods. 2. Answer C.
c. The child does not sit unsupported. In an emergency, intraosseous drug administration is typically
d. The baby cries whenever the mother goes out. used when a child is critically ill and under age 3.
44. Cherry, the mother of an 11-month-old girl, 3. Answer D.
Elizabeth, is in the clinic for her daughter’s A family’s behavioral patterns and values are passed from one
immunizations. She expresses concern to the nurse generation to the next. Cultural background commonly plays
that Elizabeth cannot yet walk. The nurse correctly a major role in determining a family’s health practices.
replies that, according to the Denver Developmental Physical characteristics do not indicate a child’s culture.
Screen, the median age for walking is: Although heritage plays a role in culture, it does not dictate a
a. 12 months. group’s shared values and its effect on culture is weaker than
b. 15 months. that of behavioral patterns.
c. 10 months. 4. Answer A.
d. 14 months. Because the anterior fontanel normally closes between ages
45. Sunshine, age 13, has had a lumbar puncture to 12 and 18 months, the nurse should notify the doctor
examine the CSF to determine if bacterial infection promptly of this finding. An open fontanel does not indicate
exists. The best position to keep her in after the abuse and is not associated with Tay-Sachs disease.
procedure is: 5. Answer B.
a. prone for two hours to prevent aspiration, should she Ulcerative colitis causes profuse diarrhea, intense abdominal
vomit.
cramps, anal fissures, and abdominal distentions are more 17. Answer D.
common in Crohn’s disease. Because adolescents absorb less information through reading,
6. Answer D. providing age-appropriate reading materials is the least
The recommended injection site for an infant is the vastus effective way to teach parenting skills to an adolescent. The
lateralis or rectus femoris muscles. The deltoid is other options engage more than one of the senses and
inappropriate. The dorsogluteal and ventrogluteal sites can be therefore serve as effective teaching strategies.
used only in toddlers who have been walking for about 1 18. Answer D.
year. Normally the top of the ear aligns with an imaginary line
7. Answer C. drawn across the inner and outer canthus of the eye. Ears set
A negative nitrogen balance may result from inadequate below this line are associated with renal anomalies or mental
protein intake and is best detected by measuring the total retardation. Low-set ears do not accompany otogenous
protein level. Measuring total iron-binding capacity, tetanus, tracheoesophageal fistula, or congenital heart
hemoglobin, and serum transferrin levels would help detect defects.
iron-deficiency anemia, not a negative nitrogen balance. 19. Answer A.
8. Answer B. At age 3, gross motor development and refinement in eye-
According to Erikson, the primary psychosocial task during hand coordination enable a child to ride a tricycle. The fine
adolescence is to establish a personal identity confusion. The motor skills required to tie shoelaces and the gross motor
adolescent attempts to establish a group identity by seeking skills requires for roller-skating and jumping rope develop
acceptance and approval from peers, and strives to attain a around age 5.
personal identity by becoming more independent from the 20. Answer A.
family. Becoming industrious is the developmental task of the In a child, Eustachian tubes are short and lie in a horizontal
school-age child, achieving intimacy is the task of the young plane, promoting entry of nasopharyngeal secretions into the
adult, and developing initiative is the task of the preschooler. tubes and thus setting the stage for otitis media. The
9. Answer B. nasopharynx, tympanic membrane, external ear canal have
Preschool-age children are most likely to view illness as a no unusual features that would predispose a child to otitis
punishment for misdeeds. Separation anxiety, although seen media.
in all age group, is most common in older infants. Fear of 21. Answer A.
death is typical of older school-age children and adolescents. Increased urine output, a sign of improving kidney function,
Adolescents also fear mutilation. typically is the first sign that a child with acute
10. Answer C. poststreptococcal glomerulonephritis (APSGN) is improving.
The nurse should obtain objective information about the Increased appetite, an increased energy level, and decreased
child’s nutritional intake, such as by asking about what the diarrhea are not specific to APSGN.
child ate for a specific meal. The other options ask for 22. Answer C.
subjective replies that would be open to interpretation. The primary purpose of administering corticosteroids to a
11. Answer A. child with nephritic syndrome is to decrease proteinuria.
The most important data to obtain on a child’s arrival in the Corticosteroids have no effect on blood pressure. Although
emergency department are vital sign measurements. The they help reduce inflammation, this is not the reason for their
nurse should gather the other data later. use in patients with nephritic syndrome. Corticosteroids may
12. Answer D. predispose a patient to infection.
The stress of starting nursery school may trigger a return to a 23. Answer A.
level of successful behavior from earlier stages of In an infant, signs of fluid volume deficit (dehydration)
development. A child’s skills remain intact, although increased include sunken fontanels, increased pulse rate, and decreased
stress may prevent the child from using these skills. Growth blood pressure. They occur when the body can no longer
occurs when the child does not regress. Parents rarely desire maintain sufficient intravascular fluid volume. When this
less mature behaviors. happens, the kidneys conserve water to minimize fluid loss,
13. Answer D. which results in concentrated urine with a high specific
A child’s poor progress in school may indicate a visual gravity.
disturbance. The other options are more appropriate 24. Answer C.
questions to ask when assessing vision in a geriatric patient. The nurse should shake a suspension before administration to
14. Answer C. dispersed drug particles uniformly. Diluting the suspension
Rice cereal is the first solid food an infant should receive and crushing particles are not recommended for this drug
because it is easy to digest and is associated with few form.
allergies. Next, the infant can receive pureed fruits, such as 25. Answer A.
bananas, applesauce, and pears, followed by pureed Fluid volume replacement must be calculated to the child’s
vegetables, egg yolks, cheese, yogurt, and finally, meat. Egg weight to avoid over-hydration. Initial fluid bolus is
whites should not be given until age 9 months because they administered at 20 ml/kg, followed by another 20 ml/kg bolus
may trigger a food allergy. if there is no improvement in fluid status.
15. Answer C. 26. Answer B.
Succinylcholine is an ultra-short-acting depolarizing agent According to the American Association on Mental Deficiency, a
used for rapid-sequence intubation. Bradycardia can occur, person with an intelligence quotient (IQ) between 50 and 70
especially in children. Atropine is the drug of choice in is classified as mildly mentally retarded but educable. One
treating succinylcholine-induced bradycardia. Lidocaine is with an IQ between 35 and 50 is classified as moderately
used in adults only. Epinephrine bolus and isoproterenol are retarded but trainable. One with an IQ below 36 is severely
not used in rapid-sequence intubation because of their and profoundly impaired, requiring custodial care.
profound cardiac effects. 27. Answer C.
16. Answer A. Role and relationship patterns focus on body image and the
Bryant’s traction is used to treat femoral fractures of patient’s relationship with others, which commonly
congenital hip dislocation in children under age 2 who weigh interrelated with food intake. Questions about activities and
less than 30 lb (13.6 kg). Buck’s extension traction is skin food preferences elicit information about health promotion
traction used for short-term immobilization or to correct bone and health protection behaviors. Questions about food
deformities or contractures; overhead suspension traction is allergies elicit information about health and illness patterns.
used to treat fractures of the humerus; and 90-90 traction is 28. Answer B.
used to treat femoral fracture in children over age 2. SIDS can occur any time between 1 week and 1 year of age.
The incidence peaks at 2 to 4 months of age.
29. Answer D. 44. Answer A.
An adolescent who demonstrates a preoccupation with death By 12 months, 50 percent of children can walk well.
(such as by talking about death frequently) should be 45. Answer C.
considered at high risk for suicide. Although depression, Lying flat keeps the patient from having a “spinal headache.”
excessive sleepiness, and a history of cocaine use may occur Increasing the fluid intake will assist in replenishing the lost
in suicidal adolescents, they also occur in adolescents who are fluid during this time.
not suicidal. 46. Answer C.
30. Answer D. The patient can lose vascular status without the nurse being
The most common sign of Wilms’ tumor is a painless, aware if left for more than 4 hours, yet checks should not be
palpable abdominal mass, sometimes accompanied by an so frequent that the patient becomes anxious. Vital signs are
increase in abdominal girth. Gross hematuria is uncommon, generally checked q4h, at which time the CSM checks can
although microscopic hematuria may be present. Dysuria is easily be performed.
not associated with Wilms’ tumor. Nausea and vomiting are 47. Answer A.
rare in children with Wilms’ tumor. Bronchodilators can produce the side effects listed in answer
31. Answer B. choice (A) for a short time after the patient begins using
When administering chemotherapy, the nurse should observe them.
for an anaphylactic reaction for 20 minutes and stop the 48. Answer D.
medication if one is suspected. Chemotherapy is associated Meningitis is an infection of the meninges, the outer
with both general and specific adverse effects, therefore close membrane of the brain. Since it is surrounded by
monitoring for them is important. cerebrospinal fluid, a lumbar puncture will help to identify the
32. Answer D. organism involved.
When examining a toddler or any small child, the best way to 49. Answer C.
perform the exam is from least to most intrusive. Starting at The glycosylated hemoglobin test measures glucose levels for
the head or abdomen is intrusive and should be avoided. the previous 3 to 4 months.
Proceeding from distal to proximal is inappropriate at any 50. Answer D.
age. Capillary refill, pulses, and skin temperature and color are
33. Answer C. indicative of intact circulation and absence of compartment
Rheumatic fever results as a delayed reaction to inadequately syndrome. Skin integrity is less important.
treated group A β-hemolytic streptococcal infection.
34. Answer A. NCLEX Exam: Maternal and Child Health Nursing 1 (30
The child with congenital heart disease develops polycythemia Items)
resulting from an inadequate mechanism to compensate for
decreased oxygen saturation 1. A postpartum patient was in labor for 30 hours and
35. Answer D. had ruptured membranes for 24 hours. For which of the
Mycostatin suspension is given as swab. Never mix following would the nurse be alert?
medications with food and formula. a. Endometritis
36. Answer C. b. Endometriosis
If the child is hungry he/she more likely would finish his c. Salpingitis
meals. Therefore, the mother should be advised not to give d. Pelvic thrombophlebitis
snacks to the child. The child is a “busy toddler.” He/she will 2. A client at 36 weeks gestation is schedule for a
not able to keep still for a long time. routine ultrasound prior to an amniocentesis. After
37. Answer D. teaching the client about the purpose for the
Normal heart rate of the newborn is 120 to 160 bpm. Choices ultrasound, which of the following client statements
A, B, and C are normal assessment findings (uneven head would indicate to the nurse in charge that the client
shape is molding). needs further instruction?
38. Answer A. a. The ultrasound will help to locate the placenta
Lead poisoning may be caused by inhalation of dusk and b. The ultrasound identifies blood flow through the umbilical
smoke from leaded gas. It may also be caused by lead-based cord
paint, soil, water (especially from plumbings of old houses). c. The test will determine where to insert the needle
39. Answer A. d. The ultrasound locates a pool of amniotic fluid
Glucose-6-phosphate dehydrogenase deficiency (G6PD) is an 3. While the postpartum client is receiving heparin for
X-linked recessive hereditary disease characterised by thrombophlebitis, which of the following drugs would
abnormally low levels of glucose-6-phosphate dehydrogenase the nurse expect to administer if the client develops
(abbreviated G6PD or G6PDH), a metabolic enzyme involved complications related to heparin therapy?
in the pentose phosphate pathway, especially important in a. Calcium gluconate
red blood cell metabolism. b. Protamine sulfate
40. Answer C. c. Methylergonovine (Methergine)
For the child with iron deficiency anemia, the blood study d. Nitrofurantoin (macrodantin)
results most likely would reveal decreased mean corpuscular 4. When caring for a 3-day-old neonate who is
volume (MCV) which demonstrates microcytic anemia, receiving phototherapy to treat jaundice, the nurse in
decreased hemoglobin, decreased hematocrit and elevated charge would expect to do which of the following?
total iron binding capacity. a. Turn the neonate every 6 hours
41. Answer D. b. Encourage the mother to discontinue breast-feeding
The primary role of the nurse when a patient has a seizure is c. Notify the physician if the skin becomes bronze in color
to protect the patient from harming him or herself. d. Check the vital signs every 2 to 4 hours
42. Answer A. 5. A primigravida in active labor is about 9 days post-
During this period, which lasts up to the age of 18-21 years, term. The client desires a bilateral pudendal block
the individual develops a sense of “self.” Peers have a major anesthesia before delivery. After the nurse explains
big influence over behavior, and the major decision is to this type of anesthesia to the client, which of the
determine a vocational goal. following locations identified by the client as the area
43. Answer C. of relief would indicate to the nurse that the teaching
Over 90% percent of babies can sit unsupported by nine was effective?
months. Most babies cannot say “mama” in the sense that it a. Back
refers to their mother at this time. b. Abdomen
c. Fundus 14. While the client is in active labor with twins and the
d. Perineum cervix is 5 cm dilates, the nurse observes contractions
6. The nurse is caring for a primigravida at about 2 occurring at a rate of every 7 to 8 minutes in a 30-
months and 1 week gestation. After explaining self- minute period. Which of the following would be the
care measures for common discomforts of pregnancy, nurse’s most appropriate action?
the nurse determines that the client understands the a. Note the fetal heart rate patterns
instructions when she says: b. Notify the physician immediately
a. “Nausea and vomiting can be decreased if I eat a few c. Administer oxygen at 6 liters by mask
crackers before arising” d. Have the client pant-blow during the contractions
b. “If I start to leak colostrum, I should cleanse my nipples 15. A client tells the nurse, “I think my baby likes to
with soap and water” hear me talk to him.” When discussing neonates and
c. “If I have a vaginal discharge, I should wear nylon stimulation with sound, which of the following would
underwear” the nurse include as a means to elicit the best
d. “Leg cramps can be alleviated if I put an ice pack on the response?
area” a. High-pitched speech with tonal variations
7. Thirty hours after delivery, the nurse in charge plans b. Low-pitched speech with a sameness of tone
discharge teaching for the client about infant care. By c. Cooing sounds rather than words
this time, the nurse expects that the phase of d. Repeated stimulation with loud sounds
postpartum psychological adaptation that the client 16. A 31-year-old multipara is admitted to the birthing
would be in would be termed which of the following? room after initial examination reveals her cervix to be
a. Taking in at 8 cm, completely effaced (100 %), and at 0 station.
b. Letting go What phase of labor is she in?
c. Taking hold a. Active phase
d. Resolution b. Latent phase
8. A pregnant client is diagnosed with partial placenta c. Expulsive phase
previa. In explaining the diagnosis, the nurse tells the d. Transitional phase
client that the usual treatment for partial placenta 17. A pregnant patient asks the nurse if she can take
previa is which of the following? castor oil for her constipation. How should the nurse
a. Activity limited to bed rest respond?
b. Platelet infusion a. “Yes, it produces no adverse effect.”
c. Immediate cesarean delivery b. “No, it can initiate premature uterine contractions.”
d. Labor induction with oxytocin c. “No, it can promote sodium retention.”
9. The nurse plans to instruct the postpartum client d. “No, it can lead to increased absorption of fat-soluble
about methods to prevent breast engorgement. Which vitamins.”
of the following measures would the nurse include in 18. A patient in her 14th week of pregnancy has
the teaching plan? presented with abdominal cramping and vaginal
a. Feeding the neonate a maximum of 5 minutes per side on bleeding for the past 8 hours. She has passed several
the first day cloth. What is the primary nursing diagnosis for this
b. Wearing a supportive brassiere with nipple shields patient?
c. Breast-feeding the neonate at frequent intervals a. Knowledge deficit
d. Decreasing fluid intake for the first 24 to 48 hours b. Fluid volume deficit
10. When the nurse on duty accidentally bumps the c. Anticipatory grieving
bassinet, the neonate throws out its arms, hands d. Pain
opened, and begins to cry. The nurse interprets this 19. Immediately after a delivery, the nurse-midwife
reaction as indicative of which of the following assesses the neonate’s head for signs of molding.
reflexes? Which factors determine the type of molding?
a. Startle reflex a. Fetal body flexion or extension
b. Babinski reflex b. Maternal age, body frame, and weight
c. Grasping reflex c. Maternal and paternal ethnic backgrounds
d. Tonic neck reflex d. Maternal parity and gravidity
11. A primigravida client at 25 weeks gestation visits 20. For a patient in active labor, the nurse-midwife
the clinic and tells the nurse that her lower back aches plans to use an internal electronic fetal monitoring
when she arrives home from work. The nurse should (EFM) device. What must occur before the internal EFM
suggest that the client perform: can be applied?
a. Tailor sitting a. The membranes must rupture
b. Leg lifting b. The fetus must be at 0 station
c. Shoulder circling c. The cervix must be dilated fully
d. Squatting exercises d. The patient must receive anesthesia
12. Which of the following would the nurse in charge 21. A primigravida patient is admitted to the labor
do first after observing a 2-cm circle of bright red delivery area. Assessment reveals that she is in early
bleeding on the diaper of a neonate who just had a part of the first stage of labor. Her pain is likely to be
circumcision? most intense:
a. Notify the neonate’s pediatrician immediately a. Around the pelvic girdle
b. Check the diaper and circumcision again in 30 minutes b. Around the pelvic girdle and in the upper arms
c. Secure the diaper tightly to apply pressure on the site c. Around the pelvic girdle and at the perineum
d. Apply gentle pressure to the site with a sterile gauze pad d. At the perineum
13. Which of the following would the nurse most likely 22. A female adult patient is taking a progestin-only
expect to find when assessing a pregnant client with oral contraceptive, or mini pill. Progestin use may
abruption placenta? increase the patient’s risk for:
a. Excessive vaginal bleeding a. Endometriosis
b. Rigid, board-like abdomen b. Female hypogonadism
c. Titanic uterine contractions c. Premenstrual syndrome
d. Premature rupture of membranes d. Tubal or ectopic pregnancy
23. A patient with pregnancy-induced hypertension Doppler imaging ultrasonography identifies blood flow through
probably exhibits which of the following symptoms? the umbilical cord. A routine ultrasound does not accomplish
a. Proteinuria, headaches, vaginal bleeding this.
b. Headaches, double vision, vaginal bleeding 3. Answer B.
c. Proteinuria, headaches, double vision Protamine sulfate is a heparin antagonist given intravenously
d. Proteinuria, double vision, uterine contractions to counteract bleeding complications cause by heparin
24. Because cervical effacement and dilation are not overdose.
progressing in a patient in labor,the doctor orders I.V. 4. Answer D.
administration of oxytocin (Pitocin). Why must the While caring for an infant receiving phototherapy for
nurse monitor the patient’s fluid intake and output treatment of jaundice, vital signs are checked every 2 to 4
closely during oxytocin administration? hours because hyperthermia can occur due to the
a. Oxytocin causes water intoxication phototherapy lights.
b. Oxytocin causes excessive thirst 5. Answer D.
c. Oxytocin is toxic to the kidneys A bilateral pudendal block is used for vaginal deliveries to
d. Oxytocin has a diuretic effect relieve pain primarily in the perineum and vagina. Pudendal
25. Five hours after birth, a neonate is transferred to block anesthesia is adequate for episiotomy and its repair.
the nursery, where the nurse intervenes to prevent 6. Answer A.
hypothermia. What is a common source of radiant heat Eating dry crackers before arising can assist in decreasing the
loss? common discomfort of nausea and vomiting. Avoiding strong
a. Low room humidity food odors and eating a high-protein snack before bedtime
b. Cold weight scale can also help.
c. Cools incubator walls 7. Answer C.
d. Cool room temperature Beginning after completion of the taking-in phase, the taking-
26. After administering bethanechol to a patient with hold phase lasts about 10 days. During this phase, the client
urine retention, the nurse in charge monitors the is concerned with her need to resume control of all facets of
patient for adverse effects. Which is most likely to her life in a competent manner. At this time, she is ready to
occur? learn self-care and infant care skills.
a. Decreased peristalsis 8. Answer A.
b. Increase heart rate Treatment of partial placenta previa includes bed rest,
c. Dry mucous membranes hydration, and careful monitoring of the client’s bleeding.
d. Nausea and Vomiting 9. Answer C.
27. The nurse in charge is caring for a patient who is in Prevention of breast engorgement is key. The best technique
the first stage of labor. What is the shortest but most is to empty the breast regularly with feeding. Engorgement is
difficult part of this stage? less likely when the mother and neonate are together, as in
a. Active phase single room maternity care continuous rooming in, because
b. Complete phase nursing can be done conveniently to meet the neonate’s and
c. Latent phase mother’s needs.
d. Transitional phase 10. Answer A.
28. After 3 days of breast-feeding, a postpartal patient The Moro, or startle, reflex occurs when the neonate responds
reports nipple soreness. To relieve her discomfort, the to stimuli by extending the arms, hands open, and then
nurse should suggest that she: moving the arms in an embracing motion. The Moro reflex,
a. Apply warm compresses to her nipples just before feedings present at birth, disappears at about age 3 months.
b. Lubricate her nipples with expressed milk before feeding 11. Answer A.
c. Dry her nipples with a soft towel after feedings Tailor sitting is an excellent exercise that helps to strengthen
d. Apply soap directly to her nipples, and then rinse the client’s back muscles and also prepares the client for the
29. The nurse is developing a teaching plan for a process of labor. The client should be encouraged to rest
patient who is 8 weeks pregnant. The nurse should tell periodically during the day and avoid standing or sitting in
the patient that she can expect to feel the fetus move one position for a long time.
at which time? 12. Answer D.
a. Between 10 and 12 weeks’ gestation If bleeding occurs after circumcision, the nurse should first
b. Between 16 and 20 weeks’ gestation apply gently pressure on the area with sterile gauze. Bleeding
c. Between 21 and 23 weeks’ gestation is not common but requires attention when it occurs.
d. Between 24 and 26 weeks’ gestation 13. Answer B.
30. Normal lochial findings in the first 24 hours post- The most common assessment finding in a client with
delivery include: abruption placenta is a rigid or boardlike abdomen. Pain,
a. Bright red blood usually reported as a sharp stabbing sensation high in the
b. Large clots or tissue fragments uterine fundus with the initial separation, also is common.
c. A foul odor 14. Answer B.
d. The complete absence of lochia The nurse should contact the physician immediately because
the client is most likely experiencing hypotonic uterine
contractions. These contractions tend to be painful but
Answers and Rationale ineffective. The usual treatment is oxytocin augmentation,
1. Answer A. unless cephalopelvic disproportion exists.
Endometritis is an infection of the uterine lining and can occur 15. Answer A.
after prolonged rupture of membranes. Endometriosis does Providing stimulation and speaking to neonates is important.
not occur after a strong labor and prolonged rupture of Some authorities believe that speech is the most important
membranes. Salpingitis is a tubal infection and could occur if type of sensory stimulation for a neonate. Neonates respond
endometritis is not treated. Pelvic thrombophlebitis involves a best to speech with tonal variations and a high-pitched voice.
clot formation but it is not a complication of prolonged A neonate can hear all sound louder than about 55 decibels.
rupture of membranes. 16. Answer D.
2. Answer B. The transitional phase of labor extends from 8 to 10 cm; it is
Before amniocentesis, a routine ultrasound is valuable in the shortest but most difficult and intense for the patient. The
locating the placenta, locating a pool of amniotic fluid, and latent phase extends from 0 to 3 cm; it is mild in nature. The
showing the physician where to insert the needle. Color active phase extends from 4 to 7 cm; it is moderate for the
patient. The expulsive phase begins immediately after the occurs during the second, not first, stage of labor. The latent
birth and ends with separation and expulsion of the placenta. phase lasts 5 to 8 hours and is marked by mild, short,
17. Answer B. irregular contractions.
Castor oil can initiate premature uterine contractions in 28. Answer B.
pregnant women. It also can produce other adverse effects, Measures that help relieve nipple soreness in a breast-feeding
but it does not promote sodium retention. Castor oils is not patient include lubrication the nipples with a few drops of
known to increase absorption of fat-soluble vitamins, expressed milk before feedings, applying ice compresses just
although laxatives in general may decrease absorption if before feeding, letting the nipples air dry after feedings, and
intestinal motility is increased. avoiding the use of soap on the nipples.
18. Answer B. 29. Answer B.
If bleeding and clots are excessive, this patient may become A pregnant woman usually can detect fetal movement
hypovolemic. Pad count should be instituted. Although the (quickening) between 16 and 20 weeks’ gestation. Before 16
other diagnoses are applicable to this patient, they are not weeks, the fetus is not developed enough for the woman to
the primary diagnosis. detect movement. After 20 weeks, the fetus continues to gain
19. Answer A. weight steadily, the lungs start to produce surfactant, the
Fetal attitude—the overall degree of body flexion or brain is grossly formed, and myelination of the spinal cord
extension—determines the type of molding in the head a begins.
neonate. Molding is not influenced by maternal age, body 30. Answer A.
frame, weight, parity, and gravidity or by maternal and Lochia should never contain large clots, tissue fragments, or
paternal ethnic backgrounds. membranes. A foul odor may signal infection, as may absence
20. Answer A. of lochia.
Internal EFM can be applied only after the patient’s
membranes have ruptured, when the fetus is at least at the - NCLEX Exam: Maternal and Child Health Nursing 2 (30
1 station, and when the cervix is dilated at least 2 cm. Items)
although the patient may receive anesthesia, it is not required
before application of an internal EFM device. 1. Accompanied by her husband, a patient seeks
21. Answer A. admission to the labor and delivery area. The client
During most of the first stage of labor, pain centers around states that she is in labor and says she attended the
the pelvic girdle. During the late part of this stage and the hospital clinic for prenatal care. Which question should
early part of the second stage, pain spreads to the upper legs the nurse ask her first?
and perineum. During the late part of the second stage and a. “Do you have any chronic illness?”
during childbirth, intense pain occurs at the perineum. Upper b. “Do you have any allergies?”
arm pain is not common during any stage of labor. c. “What is your expected due date?”
22. Answer D. d. “Who will be with you during labor?”
Women taking the minipill have a higher incidence of tubal 2. A patient is in the second stage of labor. During this
and ectopic pregnancies, possibly because progestin slows stage, how frequently should the nurse in charge
ovum transport through the fallopian tubes. Endometriosis, assess her uterine contractions?
female hypogonadism, and premenstrual syndrome are not a. Every 5 minutes
associated with progestin-only oral contraceptives. b. Every 15 minutes
23. Answer C. c. Every 30 minutes
A patient with pregnancy-induced hypertension complains of d. Every 60 minutes
headache, double vision, and sudden weight gain. A urine 3. A patient is in her last trimester of pregnancy. Nurse
specimen reveals proteinuria. Vaginal bleeding and uterine Vickie should instruct her to notify her primary health
contractions are not associated with pregnancy-induced care provider immediately if she notices:
hypertension. a. Blurred vision
24. Answer A. b. Hemorrhoids
The nurse should monitor fluid intake and output because c. Increased vaginal mucus
prolonged oxytocin infusion may cause severe water d. Shortness of breath on exertion
intoxication, leading to seizures, coma, and death. Excessive 4. The nurse in-charge is reviewing a patient’s prenatal
thirst results from the work of labor and limited oral fluid history. Which finding indicates a genetic risk factor?
intake—not oxytocin. Oxytocin has no nephrotoxic or diuretic a. The patient is 25 years old
effects. In fact, it produces an antidiuretic effect. b. The patient has a child with cystic fibrosis
25. Answer C. c. The patient was exposed to rubella at 36 weeks’ gestation
Common source of radiant heat loss includes cool incubator d. The patient has a history of preterm labor at 32 weeks’
walls and windows. Low room humidity promotes evaporative gestation
heat loss. When the skin directly contacts a cooler object, 5. A adult female patient is using the rhythm (calendar-
such as a cold weight scale, conductive heat loss may occur. basal body temperature) method of family planning. In
A cool room temperature may lead to convective heat loss. this method, the unsafe period for sexual intercourse is
26. Answer D. indicated by:
Bethanechol will increase GI motility, which may cause a. Return preovulatory basal body temperature
nausea, belching, vomiting, intestinal cramps, and diarrhea. b. Basal body temperature increase of 0.1 degrees to 0.2
Peristalsis is increased rather than decreased. With high degrees on the 2nd or 3rd day of cycle
doses of bethanechol, cardiovascular responses may include c. 3 full days of elevated basal body temperature and clear,
vasodilation, decreased cardiac rate, and decreased force of thin cervical mucus
cardiac contraction, which may cause hypotension. Salivation d. Breast tenderness and mittelschmerz
or sweating may gently increase. 6. During a nonstress test (NST), the electronic tracing
27. Answer D. displays a relatively flat line for fetal movement,
The transitional phase, which lasts 1 to 3 hours, is the making it difficult to evaluate the fetal heart rate
shortest but most difficult part of the first stage of labor. This (FHR). To mark the strip, the nurse in charge should
phase is characterized by intense uterine contractions that instruct the client to push the control button at which
occur every 1 ½ to 2 minutes and last 45 to 90 seconds. The time?
active phase lasts 4 ½ to 6 hours; it is characterized by a. At the beginning of each fetal movement
contractions that starts out moderately intense, grow b. At the beginning of each contraction
stronger, and last about 60 seconds. The complete phase
c. After every three fetal movements c. The fetus isn’t in distress at this time.
d. At the end of fetal movement d. The client should repeat the test in 1 week.
7. When evaluating a client’s knowledge of symptoms 15. A client who is 36 weeks pregnant comes to the
to report during her pregnancy, which statement would clinic for a prenatal checkup. To assess the client’s
indicate to the nurse in charge that the client preparation for parenting, the nurse might ask which
understands the information given to her? question?
a. “I’ll report increased frequency of urination.” a. “Are you planning to have epidural anesthesia?”
b. “If I have blurred or double vision, I should call the clinic b. “Have you begun prenatal classes?”
immediately.” c. “What changes have you made at home to get ready for
c. “If I feel tired after resting, I should report it immediately.” the baby?”
d. “Nausea should be reported immediately.” d. “Can you tell me about the meals you typically eat each
8. When assessing a client during her first prenatal day?”
visit, the nurse discovers that the client had a reduction 16. A client who’s admitted to labor and delivery has
mammoplasty. The mother indicates she wants to the following assessment findings: gravida 2 para 1,
breast-feed. What information should the nurse give to estimated 40 weeks gestation, contractions 2 minutes
this mother regarding breastfeeding success? apart, lasting 45 seconds, vertex +4 station. Which of
a. “It’s contraindicated for you to breastfeed following this the following would be the priority at this time?
type of surgery.” a. Placing the client in bed to begin fetal monitoring.
b. “I support your commitment; however, you may have to b. Preparing for immediate delivery.
supplement each feeding with formula.” c. Checking for ruptured membranes.
c. “You should check with your surgeon to determine whether d. Providing comfort measures.
breast-feeding would be possible.” 17. The nurse is caring for a client in labor. The
d. “You should be able to breastfeed without difficulty.” external fetal monitor shows a pattern of variable
9. Following a precipitous delivery, examination of the decelerations in fetal heart rate. What should the nurse
client’s vagina reveals a fourth-degree laceration. do first?
Which of the following would be contraindicated when a. Change the client’s position.
caring for this client? b. Prepare for emergency cesarean section.
a. Applying cold to limit edema during the first 12 to 24 hours c. Check for placenta previa.
b. Instructing the client to use two or more peri pads to d. Administer oxygen.
cushion the area 18. The nurse in charge is caring for a postpartum
c. Instructing the client on the use of sitz baths if ordered client who had a vaginal delivery with a midline
d. Instructing the client about the importance of perineal episiotomy. Which nursing diagnosis takes priority for
(Kegel) exercises this client?
10. A client makes a routine visit to the prenatal clinic. a. Risk for deficient fluid volume related to hemorrhage
Although she is 14 weeks pregnant, the size of her b. Risk for infection related to the type of delivery
uterus approximates that in an 18- to 20-week c. Pain related to the type of incision
pregnancy. Dr. Charles diagnoses gestational d. Urinary retention related to periurethral edema
trophoblastic disease and orders ultrasonography. The 19. Which change would the nurse identify as a
nurse expects ultrasonography to reveal: progressive physiological change in postpartum period?
a. an empty gestational sac. a. Lactation
b. grapelike clusters. b. Lochia
c. a severely malformed fetus. c. Uterine involution
d. an extrauterine pregnancy. d. Diuresis
11. After completing a second vaginal examination of a 20. A 39-year-old at 37 weeks gestation is admitted to
client in labor, the nurse-midwife determines that the the hospital with complaints of vaginal bleeding
fetus is in the right occiput anterior position and at (– following the use of cocaine 1 hour earlier. Which
1) station. Based on these findings, the nurse-midwife complication is most likely causing the client’s
knows that the fetal presenting part is: complaint of vaginal bleeding?
a. 1 cm below the ischial spines. a. Placenta previa
b. directly in line with the ischial spines. b. Abruptio placentae
c. 1 cm above the ischial spines. c. Ectopic pregnancy
d. in no relationship to the ischial spines. d. Spontaneous abortion
12. Which of the following would be inappropriate to 21. A client with type 1 diabetes mellitus who is a
assess in a mother who’s breastfeeding? multigravida visits the clinic at 27 weeks gestation. The
a. The attachment of the baby to the breast. nurse should instruct the client that for most pregnant
b. The mother’s comfort level with positioning the baby. women with type 1 diabetes mellitus:
c. Audible swallowing. a. Weekly fetal movement counts are made by the mother.
d. The baby’s lips smacking b. Contraction stress testing is performed weekly.
13. During a prenatal visit at 4 months gestation, a c. Induction of labor is begun at 34 weeks’ gestation.
pregnant client asks whether tests can be done to d. Nonstress testing is performed weekly until 32 weeks’
identify fetal abnormalities. Between 18 and 40 weeks gestation
gestation, which procedure is used to detect fetal 22. When administering magnesium sulfate to a client
anomalies? with preeclampsia, the nurse understands that this
a. Amniocentesis. drug is given to:
b. Chorionic villi sampling. a. Prevent seizures
c. Fetoscopy. b. Reduce blood pressure
d. Ultrasound c. Slow the process of labor
14. A client, 30 weeks pregnant, is scheduled for a d. Increase dieresis
biophysical profile (BPP) to evaluate the health of her 23. What is the approximate time that the blastocyst
fetus. Her BPP score is 8. What does this score spends traveling to the uterus for implantation?
indicate? a. 2 days
a. The fetus should be delivered within 24 hours. b. 7 days
b. The client should repeat the test in 24 hours. c. 10 days
d. 14 weeks
24. After teaching a pregnant woman who is in labor 3. Answer A.
about the purpose of the episiotomy, which of the Blurred vision or other visual disturbance, excessive weight
following purposes stated by the client would indicate gain, edema, and increased blood pressure may signal severe
to the nurse that the teaching was effective? preeclampsia. This condition may lead to eclampsia, which
a. Shortens the second stage of labor has potentially serious consequences for both the patient and
b. Enlarges the pelvic inlet fetus. Although hemorrhoids may be a problem during
c. Prevents perineal edema pregnancy, they do not require immediate attention.
d. Ensures quick placenta delivery Increased vaginal mucus and dyspnea on exertion are
25. A primigravida client at about 35 weeks gestation expected as pregnancy progresses.
in active labor has had no prenatal care and admits to 4. Answer B.
cocaine use during the pregnancy. Which of the Cystic fibrosis is a recessive trait; each offspring has a one in
following persons must the nurse notify? four chance of having the trait or the disorder. Maternal age
a. Nursing unit manager so appropriate agencies can be is not a risk factor until age 35, when the incidence of
notified chromosomal defects increases. Maternal exposure to rubella
b. Head of the hospital’s security department during the first trimester may cause congenital defects.
c. Chaplain in case the fetus dies in utero Although a history or preterm labor may place the patient at
d. Physician who will attend the delivery of the infant risk for preterm labor, it does not correlate with genetic
26. When preparing a teaching plan for a client who is defects.
to receive a rubella vaccine during the postpartum 5. Answer C.
period, the nurse in charge should include which of the Ovulation (the period when pregnancy can occur) is
following? accompanied by a basal body temperature increase of 0.7
a. The vaccine prevents a future fetus from developing degrees F to 0.8 degrees F and clear, thin cervical mucus. A
congenital anomalies return to the preovulatory body temperature indicates a safe
b. Pregnancy should be avoided for 3 months after the period for sexual intercourse. A slight rise in basal
immunization temperature early in the cycle is not significant. Breast
c. The client should avoid contact with children diagnosed tenderness and mittelschmerz are not reliable indicators of
with rubella ovulation.
d. The injection will provide immunity against the 7-day 6. Answer A.
measles. An NST assesses the FHR during fetal movement. In a healthy
27. A client with eclampsia begins to experience a fetus, the FHR accelerates with each movement. By pushing
seizure. Which of the following would the nurse in the control button when a fetal movement starts, the client
charge do first? marks the strip to allow easy correlation of fetal movement
a. Pad the side rails with the FHR. The FHR is assessed during uterine contractions
b. Place a pillow under the left buttock in the oxytocin contraction test, not the NST. Pushing the
c. Insert a padded tongue blade into the mouth control button after every three fetal movements or at the
d. Maintain a patent airway end of fetal movement wouldn’t allow accurate comparison of
28. While caring for a multigravida client in early labor fetal movement and FHR changes.
in a birthing center, which of the following foods would 7. Answer B.
be best if the client requests a snack? Blurred or double vision may indicate hypertension or
a. Yogurt preeclampsia and should be reported immediately. Urinary
b. Cereal with milk frequency is a common problem during pregnancy caused by
c. Vegetable soup increased weight pressure on the bladder from the uterus.
d. Peanut butter cookies Clients generally experience fatigue and nausea during
29. The multigravida mother with a history of rapid pregnancy.
labor who us in active labor calls out to the nurse, “The 8. Answer B.
baby is coming!” which of the following would be the Recent breast reduction surgeries are done in a way to
nurse’s first action? protect the milk sacs and ducts, so breast-feeding after
a. Inspect the perineum surgery is possible. Still, it’s good to check with the surgeon
b. Time the contractions to determine what breast reduction procedure was done.
c. Auscultate the fetal heart rate There is the possibility that reduction surgery may have
d. Contact the birth attendant decreased the mother’s ability to meet all of her baby’s
30. While assessing a primipara during the immediate nutritional needs, and some supplemental feeding may be
postpartum period, the nurse in charge plans to use required. Preparing the mother for this possibility is extremely
both hands to assess the client’s fundus to: important because the client’s psychological adaptation to
a. Prevent uterine inversion mothering may be dependent on how successfully she breast-
b. Promote uterine involution feeds.
c. Hasten the puerperium period 9. Answer B.
d. Determine the size of the fundus Using two or more peripads would do little to reduce the pain
or promote perineal healing. Cold applications, sitz baths, and
Answers and Rationale Kegel exercises are important measures when the client has a
1. Answer C. fourth-degree laceration.
When obtaining the history of a patient who may be in labor, 10. Answer B.
the nurse’s highest priority is to determine her current status, In a client with gestational trophoblastic disease, an
particularly her due date, gravidity, and parity. Gravidity and ultrasound performed after the 3rd month shows grapelike
parity affect the duration of labor and the potential for labor clusters of transparent vesicles rather than a fetus. The
complications. Later, the nurse should ask about chronic vesicles contain a clear fluid and may involve all or part of the
illness, allergies, and support persons. decidual lining of the uterus. Usually no embryo (and
2. Answer B. therefore no fetus) is present because it has been absorbed.
During the second stage of labor, the nurse should assess the Because there is no fetus, there can be no extrauterine
strength, frequency, and duration of contraction every 15 pregnancy. An extrauterine pregnancy is seen with an ectopic
minutes. If maternal or fetal problems are detected, more pregnancy.
frequent monitoring is necessary. An interval of 30 to 60 11. Answer C.
minutes between assessments is too long because of Fetal station — the relationship of the fetal presenting part to
variations in the length and duration of patient’s labor. the maternal ischial spines — is described in the number of
centimeters above or below the spines. A presenting part 22. Answer A.
above the ischial spines is designated as –1, –2, or –3. A The chemical makeup of magnesium is similar to that of
presenting part below the ischial spines, as +1, +2, or +3. calcium and, therefore, magnesium will act like calcium in the
12. Answer D. body. As a result, magnesium will block seizure activity in a
Assessing the attachment process for breast-feeding should hyper stimulated neurologic system by interfering with signal
include all of the answers except the smacking of lips. A baby transmission at the neuromascular junction.
who’s smacking his lips isn’t well attached and can injure the 23. Answer B.
mother’s nipples. The blastocyst takes approximately 1 week to travel to the
13. Answer D. uterus for implantation.
Ultrasound is used between 18 and 40 weeks’ gestation to 24. Answer A.
identify normal fetal growth and detect fetal anomalies and An episiotomy serves several purposes. It shortens the
other problems. Amniocentesis is done during the third second stage of labor, substitutes a clean surgical incision for
trimester to determine fetal lung maturity. Chorionic villi a tear, and decreases undue stretching of perineal muscles.
sampling is performed at 8 to 12 weeks’ gestation to detect An episiotomy helps prevent tearing of the rectum but it does
genetic disease. Fetoscopy is done at approximately 18 not necessarily relieves pressure on the rectum. Tearing may
weeks’ gestation to observe the fetus directly and obtain a still occur.
skin or blood sample. 25. Answer D.
14. Answer C. The fetus of a cocaine-addicted mother is at risk for hypoxia,
The BPP evaluates fetal health by assessing five variables: meconium aspiration, and intrauterine growth retardation
fetal breathing movements, gross body movements, fetal (IUGR). Therefore, the nurse must notify the physician of the
tone, reactive fetal heart rate, and qualitative amniotic fluid client’s cocaine use because this knowledge will influence the
volume. A normal response for each variable receives 2 care of the client and neonate. The information is used only in
points; an abnormal response receives 0 points. A score relation to the client’s care.
between 8 and 10 is considered normal, indicating that the 26. Answer B.
fetus has a low risk of oxygen deprivation and isn’t in After administration of rubella vaccine, the client should be
distress. A fetus with a score of 6 or lower is at risk for instructed to avoid pregnancy for at least 3 months to prevent
asphyxia and premature birth; this score warrants detailed the possibility of the vaccines toxic effects to the fetus.
investigation. The BPP may or may not be repeated if the 27. Answer D.
score isn’t within normal limits. The priority for the pregnant client having a seizure is to
15. Answer C. maintain a patent airway to ensure adequate oxygenation to
During the third trimester, the pregnant client typically the mother and the fetus. Additionally, oxygen may be
perceives the fetus as a separate being. To verify that this administered by face mask to prevent fetal hypoxia.
has occurred, the nurse should ask whether she has made 28. Answer A.
appropriate changes at home such as obtaining infant In some birth settings, intravenous therapy is not used with
supplies and equipment. The type of anesthesia planned low-risk clients. Thus, clients in early labor are encouraged to
doesn’t reflect the client’s preparation for parenting. The eat healthy snacks and drink fluid to avoid dehydration.
client should have begun prenatal classes earlier in the Yogurt, which is an excellent source of calcium and riboflavin,
pregnancy. The nurse should have obtained dietary is soft and easily digested. During pregnancy, gastric
information during the first trimester to give the client time to emptying time is delayed. In most hospital settings, clients
make any necessary changes. are allowed only ice chips or clear liquids.
16. Answer B. 29. Answer A.
This question requires an understanding of station as part of When the client says the baby is coming, the nurse should
the intrapartum assessment process. Based on the client’s first inspect the perineum and observe for crowning to
assessment findings, this client is ready for delivery, which is validate the client’s statement. If the client is not delivering
the nurse’s top priority. Placing the client in bed, checking for precipitously, the nurse can calm her and use appropriate
ruptured membranes, and providing comfort measures could breathing techniques.
be done, but the priority here is immediate delivery. 30. Answer A.
17. Answer A. Using both hands to assess the fundus is useful for
Variable decelerations in fetal heart rate are an ominous sign, preventing uterine inversion.
indicating compression of the umbilical cord. Changing the
client’s position from supine to side-lying may immediately
correct the problem. An emergency cesarean section is
necessary only if other measures, such as changing position
and amnioinfusion with sterile saline, prove unsuccessful.
Administering oxygen may be helpful, but the priority is to
change the woman’s position and relieve cord compression.
18. Answer A.
Hemorrhage jeopardizes the client’s oxygen supply — the first
priority among human physiologic needs. Therefore, the
nursing diagnosis of Risk for deficient fluid volume related to
hemorrhage takes priority over diagnoses of Risk for
infection, Pain, and Urinary retention.
19. Answer A.
Lactation is an example of a progressive physiological change
that occurs during the postpartum period.
20. Answer B.
The major maternal adverse reactions from cocaine use in
pregnancy include spontaneous abortion first, not third,
trimester abortion and abruptio placentae.
21. Answer D.
For most clients with type 1 diabetes mellitus, nonstress
testing is done weekly until 32 weeks’ gestation and twice a
week to assess fetal well-being.

You might also like